test 2 med surg cardiac, dysrhythmias, pain and hematology

Réussis tes devoirs et examens dès maintenant avec Quizwiz!

4. A client has a platelet count of 25,000/mm3. What actions does the nurse delegate to the unlicensed assistive personnel (UAP)? (Select all that apply.) a. Assist with oral hygiene using a firm toothbrush. b. Give the client an enema if he or she is constipated. c. Help the client choose soft foods from the menu. d. Shave the male client with an electric razor. e. Use a lift sheet when needed to re-position the client.

4. A client has a platelet count of 25,000/mm3. What actions does the nurse delegate to the unlicensed assistive personnel (UAP)? (Select all that apply.) a. Assist with oral hygiene using a firm toothbrush. b. Give the client an enema if he or she is constipated. c. Help the client choose soft foods from the menu. d. Shave the male client with an electric razor. e. Use a lift sheet when needed to re-position the client.

26. A client has frequent hospitalizations for leukemia and is worried about functioning as a parent to four small children. What action by the nurse would be most helpful? a. Assist the client to make sick day plans for household responsibilities. b. Determine if there are family members or friends who can help the client. c. Help the client inform friends and family that they will have to help out. d. Refer the client to a social worker in order to investigate respite child care.

ANS: A While all options are reasonable choices, the best option is to help the client make sick day plans, as that is more comprehensive and inclusive than the other options, which focus on a single item.

18. A nurse is caring for a client who is intubated and has an intra-aortic balloon pump. The client is restless and agitated. What action should the nurse perform first for comfort? a. Allow family members to remain at the bedside. b. Ask the family if the client would like a fan in the room. c. Keep the television tuned to the clients favorite channel. d. Speak loudly to the client in case of hearing problems

ANS: A Allowing the family to remain at the bedside can help calm the client with familiar voices (and faces if the client wakes up). A fan might be helpful but may also spread germs through air movement. The TV should not be kept on all the time to allow for rest. Speaking loudly may agitate the client more.

8. A client is having a bone marrow biopsy and is extremely anxious. What action by the nurse is best? a. Assess client fears and coping mechanisms. b. Reassure the client this is a common test. c. Sedate the client prior to the procedure. d. Tell the client he or she will be asleep.

ANS: A Assessing the clients specific fears and coping mechanisms helps guide the nurse in providing holistic care that best meets the clients needs. Reassurance will be helpful but is not the best option. Sedation is usually used. The client may or may not be totally asleep during the procedure.

10. A patient is taking clopidogrel bisulfate (Plavix). When teaching this patient about dietary restrictions while taking this medication, the nurse will instruct the patient to avoid excessive consumption of which food? a. Garlic b. Grapefruit c. Green, leafy vegetables d. Red meats

ANS: A Patients taking this drug may experience increased bleeding when taken with garlic. There is no restriction for grapefruit as there is with many other medications. Green, leafy vegetables should be restricted in patients taking warfarin. Red meats are not contraindicated.

3. A nurse is assessing clients on a medical-surgical unit. Which client should the nurse identify as being at greatest risk for atrial fibrillation? a. A 45-year-old who takes an aspirin daily b. A 50-year-old who is post coronary artery bypass graft surgery c. A 78-year-old who had a carotid endarterectomy d. An 80-year-old with chronic obstructive pulmonary disease

ANS: B Atrial fibrillation occurs commonly in clients with cardiac disease and is a common occurrence after coronary artery bypass graft surgery. The other conditions do not place these clients at higher risk for atrial fibrillation.

3. A nursing student learns that many drugs can impair the immune system. Which drugs does this include? (Select all that apply.) a. Acetaminophen (Tylenol) b. Amphotericin B (Fungizone) c. Ibuprofen (Motrin) d. Metformin (Glucophage) e. Nitrofurantoin (Macrobid)

ANS: B, C, E Amphotericin B, ibuprofen, and nitrofurantoin all can disrupt the hematologic (immune) system. Acetaminophen and metformin do not

6. A nurse administers prescribed adenosine (Adenocard) to a client. Which response should the nurse assess for as the expected therapeutic response? a. Decreased intraocular pressure b. Increased heart rate c. Short period of asystole d. Hypertensive crisis

ANS: C Clients usually respond to adenosine with a short period of asystole, bradycardia, hypotension, dyspnea, and chest pain. Adenosine has no conclusive impact on intraocular pressure

16. The nurse asks a client who has experienced ventricular dysrhythmias about substance abuse. The client asks, Why do you want to know if I use cocaine? How should the nurse respond? a. Substance abuse puts clients at risk for many health issues. b. The hospital requires that I ask you about cocaine use. c. Clients who use cocaine are at risk for fatal dysrhythmias. d. We can provide services for cessation of substance abuse.

ANS: C Clients who use cocaine or illicit inhalants are particularly at risk for potentially fatal dysrhythmias. The other responses do not adequately address the clients question

22. A client has a sickle cell crisis with extreme lower extremity pain. What comfort measure does the nurse delegate to the unlicensed assistive personnel (UAP)? a. Apply ice packs to the clients legs. b. Elevate the clients legs on pillows. c. Keep the lower extremities warm. d. Place elastic bandage wraps on the clients legs.

ANS: C During a sickle cell crisis, the tissue distal to the occlusion has decreased blood flow and ischemia, leading to pain. Due to decreased blood flow, the clients legs will be cool or cold. The UAP can attempt to keep the clients legs warm. Ice and elevation will further decrease perfusion. Elastic bandage wraps are not indicated and may constrict perfusion in the legs.

15. A client with multiple myeloma demonstrates worsening bone density on diagnostic scans. About what drug does the nurse plan to teach this client? a. Bortezomib (Velcade) b. Dexamethasone (Decadron) c. Thalidomide (Thalomid) d. Zoledronic acid (Zometa)

ANS: D All the options are drugs used to treat multiple myeloma, but the drug used specifically for bone manifestations is zoledronic acid (Zometa), which is a bisphosphonate. This drug class inhibits bone resorption and is used to treat osteoporosis as well.

9. A client is having a radioisotopic imaging scan. What action by the nurse is most important? a. Assess the client for shellfish allergies. b. Place the client on radiation precautions. c. Sedate the client before the scan. d. Teach the client about the procedure.

ANS: D The nurse should ensure that teaching is done and the client understands the procedure. Contrast dye is not used, so shellfish/iodine allergies are not related. The client will not be radioactive and does not need radiation precautions. Sedation is not used in this procedure.

5. A client undergoing hemodynamic monitoring after a myocardial infarction has a right atrial pressure of 0.5 mm Hg. What action by the nurse is most appropriate? a. Level the transducer at the phlebostatic axis. b. Lay the client in the supine position. c. Prepare to administer diuretics. d. Prepare to administer a fluid bolus

ANS: D Normal right atrial pressures are from 1 to 8 mm Hg. Lower pressures usually indicate hypovolemia, so the nurse should prepare to administer a fluid bolus. The transducer should remain leveled at the phlebostatic axis. Positioning may or may not influence readings. Diuretics would be contraindicated.

4. A client is 1 day postoperative after a coronary artery bypass graft. What nonpharmacologic comfort measures does the nurse include when caring for this client? (Select all that apply.) a. Administer pain medication before ambulating. b. Assist the client into a position of comfort in bed. c. Encourage high-protein diet selections. d. Provide complementary therapies such as music. e. Remind the client to splint the incision when coughing.

ANS:B, D, E Nonpharmacologic comfort measures can include positioning, complementary therapies, and splinting the chest incision. Medications are not nonpharmacologic. Food choices are not comfort measures.

A client with diabetic neuropathy reports a burning, electrical-type in the lower extremities that is not responding to NSAIDs. You anticipate that the physician will order which adjuvant medication for this type of pain? a.Amitriptyline (Elavil) b.Corticosteroids c.Methylphenidate (Ritalin) d.Lorazepam (Ativan)

a.Amitriptyline (Elavil) Antidepressants such as amitriptyline can be given for diabetic neuropathy. Corticosteroids are for pain associated with inflammation. Methylphenidate is given to counteract sedation if the client is on opioids. Lorazepam is an anxiolytic.

3 . A nurse is teaching a client with premature ectopic beats. Which education should the nurse include in this clients teaching? (Select all that apply .) a. Smoking cessation b . Stress reduction and management c. Avoiding vagal stimulation d . Adverse effects of medications e. Foods high in potassium

ANS : A , B , D A client who has premature beats or ectopic rhythms should be taught to stop smoking , manage stress, take medications as prescribed , and report adverse effects of medications. C lients w ith premature beats are not a t risk for vasovagal attacks or potassium imbalances.

6. A nurse cares for a client who is recovering from a right-sided heart catheterization. For which complications of this procedure should the nurse assess? (Select all that apply.) a. Thrombophlebitis b. Stroke c. Pulmonary embolism d . Myocardial infarction e. Cardiac tamponade

ANS : A , C , E Complications from a right-sided heart catheterization include thrombophlebitis, pulmonary embolism , and vagal response. Cardiac tamponade is a risk of both right- and left-sided heart catheterizations. Stroke and myocardial infarction are complications of left-sided heart catheterizations.

17. A nurse is caring for four clients. Which client should the nurse assess first? a. Client with an acute myocardial infarction, pulse 102 beats/min b. Client who is 1 hour post angioplasty, has tongue swelling and anxiety c. Client who is post coronary artery bypass, chest tube drained 100 mL/hr d. Client who is post coronary artery bypass, potassium 4.2 mEq/L

ANS B The post-angioplasty client with tongue swelling and anxiety is exhibiting manifestations of an allergic reaction that could progress to anaphylaxis. The nurse should assess this client first. The client with a heart rate of 102 beats/min may have increased oxygen demands but is just over the normal limit for heart rate. The two post coronary artery bypass clients are stable.

23.A nurse is caring for a group of patients. Which patient will the nurse see first? a. A patient who received morphine and has a pulse of 62 beats/min, respirations 10 breaths/min, and blood pressure 110/60 mm Hg b. A patient lying very still in bed who reports no pain but is pale with warm, dry skin c. A patient with severe pain who is nauseated and feels like he or she is about to vomit d. A patient writhing and moaning from abdominal pain after abdominal surgery

ANS: A A respiratory rate of 10 indicates respiratory depression. A rare adverse effect of opioids in opioid-naïve patients (patients who have used opioids around the clock for less than approximately 1 week) is respiratory depression. Naloxone (Narcan) may be administered. While the other patients are experiencing pain and do need to be seen, they are not the priority since respirations are not affected

14. A client has been admitted after sustaining a humerus fracture that occurred when picking up the family cat. What test result would the nurse correlate to this condition? a. Bence-Jones protein in urine b. Epstein-Barr virus: positive c. Hemoglobin: 18 mg/dL d. Red blood cell count: 8.2/mm3

ANS: A This client has possible multiple myeloma. A positive Bence-Jones protein finding would correlate with this condition. The Epstein-Barr virus is a herpesvirus that causes infectious mononucleosis and some cancers. A hemoglobin of 18 mg/dL is slightly high for a male and somewhat high for a female; this can be caused by several conditions, and further information would be needed to correlate this value with a specific medical condition. A red blood cell count of 8.2/mm3 is also high, but again, more information would be needed to correlate this finding with a specific medical condition.

2. A patient has a serum cholesterol level of 270 mg/dL. The patient asks the nurse what this level means. Which response by the nurse is correct? a. You have a high risk for coronary artery disease. b. You have a moderate risk for coronary artery disease. c. You have a low risk for coronary artery disease. d. You have no risk for coronary artery disease.

ANS: A A value of 270 mg/dL for serum cholesterol puts the patient at high risk.

4. A client presents to the emergency department in sickle cell crisis. What intervention by the nurse takes priority? a. Administer oxygen. b. Apply an oximetry probe. c. Give pain medication. d. Start an IV line.

ANS: A All actions are appropriate, but remembering the ABCs, oxygen would come first. The main problem in a sickle cell crisis is tissue and organ hypoxia, so providing oxygen helps halt the process.

1. A nurse caring for a client with sickle cell disease (SCD) reviews the clients laboratory work. Which finding should the nurse report to the provider? a. Creatinine: 2.9 mg/dL b. Hematocrit: 30% c. Sodium: 147 mEq/L d. White blood cell count: 12,000/mm3

ANS: A An elevated creatinine indicates kidney damage, which occurs in SCD. A hematocrit level of 30% is an expected finding, as is a slightly elevated white blood cell count. A sodium level of 147 mEq/L, although slightly high, is not concerning.

14. A nurse assesses a client with tachycardia. Which clinical manifestation requires immediate intervention by the nurse? a. Mid-sternal chest pain b. Increased urine output c. Mild orthostatic hypotension d. P wave touching the T wave

ANS: A Chest pain, possibly angina, indicates that tachycardia may be increasing the clients myocardial workload and oxygen demand to such an extent that normal oxygen delivery cannot keep pace. This results in myocardial hypoxia and pain. Increased urinary output and mild orthostatic hypotension are not life-threatening conditions and therefore do not require immediate intervention. The P wave touching the T wave indicates significant tachycardia and should be assessed to determine the underlying rhythm and cause; this is an important assessment but is not as critical as chest pain, which indicates cardiac cell death.

26.The nurse is teaching a student nurse about pain assessment scales. Which statement by the student indicates effective teaching? a. "You cannot use a pain scale to compare the pain of my patient with the pain of your patient." b. "When patients say they don't need pain medication, they aren't in pain." c. "A patient's behavior is more reliable than the patient's report of pain." d. "Pain assessment scales determine the quality of a patient's pain."

ANS: A Do not use a pain scale to compare the pain of one patient to that of another. Pain is subjective and cannot be compared to the pain of another patient. Some patients do not express their pain (stoic) or do not wish to take medications to relieve the pain. This does not mean they aren't in pain. A patient's behavior is not more reliable than the patient's report of pain. Pain scales help determine severity or intensity, not quality.

5. The nurse is caring for a postoperative older patient who received PO hydrocodone with acetaminophen (Lortab) 45 minutes prior after reporting a pain level of 8 on a scale of 1 to 10. The patient reports a pain level of 4, and the nurse notes a respiratory rate of 20 breaths per minute, a heart rate of 92 beats per minute, and a blood pressure of 170/95 mm Hg. Which action will the nurse take? a. Contact the provider and request an order for a more potent opioid analgesic. b. Reassess the patient in 30 minutes. c. Request an order for ibuprofen to augment the opioid analgesic. d. Suggest that the patient use nonpharmacologic measures to relieve pain.

ANS: A Even though the patient reports decreased pain, the patients vital signs indicate continued discomfort. The nurse should contact the provider to request a stronger analgesic. The pain medication should have been effective within 30 minutes. Ibuprofen is used for musculoskeletal pain. Nonpharmacologic measures may be useful, but the patient still needs a stronger analgesic.

15. A nurse teaches a client who experiences occasional premature atrial contractions (PACs) accompanied by palpitations that resolve spontaneously without treatment. Which statement should the nurse include in this clients teaching? a. Minimize or abstain from caffeine. b. Lie on your side until the attack subsides. c. Use your oxygen when you experience PACs. d. Take amiodarone (Cordarone) daily to prevent PACs.

ANS: A PACs usually have no hemodynamic consequences. For a client experiencing infrequent PACs, the nurse should explore possible lifestyle causes, such as excessive caffeine intake and stress. Lying on the side will not prevent or resolve PACs. Oxygen is not necessary. Although medications may be needed to control symptomatic dysrhythmias, for infrequent PACs, the client first should try lifestyle changes to control them

22.The nurse is caring for a 4-year-old child who has pain. Which technique will the nurse use tobest assess pain in this child? a. Use the FACES scale. b. Check to see what previous nurses have charted. c. Ask the parents if they think their child is in pain. d. Have the child rate the level of pain on a 0 to 10 pain scale.

ANS: A The FACES scale assesses pain in children who are verbal. Because a 4-year-old is verbal, this is an appropriate scale to use with this child. Assessing pain intensity in children requires special techniques. Young children often have difficulty expressing their pain. Parents' statement of pain is not an effective way to assess pain in children because children's statements are the most important. The 0 to 10 pain scale is too difficult for a 4-year-old child to understand. Previous documentation by nurses will tell you what the child's pain has been but will not tell you the child's current pain intensity.

5. The nurse is caring for a patient who is receiving warfarin (Coumadin) and notes bruising and petechiae on the patients extremities. The nurse will request an order for which laboratory test? a. International normalized ratio (INR) b. Platelet level c. Partial thromboplastin time (PTT) and activated partial thromboplastin time (aPTT) d. Vitamin K level

ANS: A The INR is the test used most frequently to report prothrombin time results in patients taking warfarin. Warfarin is not an antiplatelet drug, so platelet levels are not indicated. PTT and aPTT are used to monitor heparin therapy. Vitamin K is an antidote for warfarin; levels are not routinely checked.

25. A nurse is caring for four clients with leukemia. After hand-off report, which client should the nurse see first? a. Client who had two bloody diarrhea stools this morning b. Client who has been premedicated for nausea prior to chemotherapy c. Client with a respiratory rate change from 18 to 22 breaths/min d. Client with an unchanged lesion to the lower right lateral malleolus

ANS: A The client who had two bloody diarrhea stools that morning may be hemorrhaging in the gastrointestinal (GI) tract and should be assessed first. The client with the change in respiratory rate may have an infection or worsening anemia and should be seen next. The other two clients are not a priority at this time.

13. A nurse prepares to discharge a client with cardiac dysrhythmia who is prescribed home health care services. Which priority information should be communicated to the home health nurse upon discharge? a. Medication reconciliation b. Immunization history c. Religious beliefs d. Nutrition preferences

ANS: A The home health nurse needs to know current medications the client is taking to ensure assessment, evaluation, and further education related to these medications. The other information will not assist the nurse to develop a plan of care for the client.

2. A nursing student asks why the anticoagulant heparin is given to patients who have disseminated intravascular coagulation (DIC) and are at risk for excessive bleeding. The nurse will explain that heparin is used in this case for which reason? a. To decrease the risk of venous thrombosis b. To dissolve blood clots as they form c. To enhance the formation of fibrous clots d. To preserve platelet function

ANS: A The primary use of heparin for patients with DIC is to prevent venous thrombosis, which can lead to pulmonary embolism or stroke. Heparin does not break down blood clots, enhance the formation of fibrous clots, or preserve platelet function.

4. A nurse is assessing a dark-skinned client for pallor. What action is best? a. Assess the conjunctiva of the eye. b. Have the client open the hand widely. c. Look at the roof of the clients mouth. d. Palpate for areas of mild swelling.

ANS: A To assess pallor in dark-skinned people, assess the conjunctiva of the eye or the mucous membranes. Looking at the roof of the mouth can reveal jaundice. Opening the hand widely is not related to pallor, nor is palpating for mild swelling.

17. A nurse supervises an unlicensed assistive personnel (UAP) applying electrocardiographic monitoring. Which statement should the nurse provide to the UAP related to this procedure? a. Clean the skin and clip hairs if needed. b. Add gel to the electrodes prior to applying them. c. Place the electrodes on the posterior chest. d. Turn off oxygen prior to monitoring the client.

ANS: A To ensure the best signal transmission, the skin should be clean and hairs clipped. Electrodes should be placed on the anterior chest, and no additional gel is needed. Oxygen has no impact on electrocardiographic monitoring

13. The nurse is caring for a patient who was admitted with a fractured leg and for observation of a closed head injury after a motor vehicle accident. The patient reports having pain at a level of 3 on a 1 to 10 pain scale. The nurse will expect the provider to order which analgesic medication for this patient? a. Acetaminophen (Tylenol) PO b. Hydromorphone HCl (Dilaudid) IM c. Morphine sulfate PCA d. Transdermal fentanyl (Duragesic)

ANS: A Use of opioid analgesics is contraindicated for patients with head injuries because of the risk of increased intracranial pressure. If opioids are necessary because of severe pain, they must be given in reduced doses. This patient is experiencing mild pain, so acetaminophen is an appropriate analgesic.

1. A nurse assesses a client who had a myocardial infarction and is hypotensive. Which additional assessment finding should the nurse expect? a. Heart rate of 120 beats/min b. Cool, clammy skin c. Oxygen saturation of 90% d. Respiratory rate of 8 breaths/min

ANS: A When a client experiences hypotension, baroreceptors in the aortic arch sense a pressure decrease in the vessels. The parasympathetic system responds by lessening the inhibitory effect on the sinoatrial node. This results in an increase in heart rate and respiratory rate. This tachycardia is an early response and is seen even when blood pressure is not critically low. An increased heart rate and respiratory rate will compensate for the low blood pressure and maintain oxygen saturations and perfusion. The client may not be able to compensate for long, and decreased oxygenation and cool, clammy skin will occur later.

8. The nurse is assessing a patient who takes warfarin (Coumadin). The nurse notes a heart rate of 92 beats per minute and a blood pressure of 88/78 mm Hg. To evaluate the reason for these vital signs, the nurse will assess the patients a. gums, nose, and skin. b. lung sounds and respiratory effort. c. skin turgor and oral mucous membranes. d. urine output and level of consciousness.

ANS: A An increased heart rate followed by a decreased systolic pressure can indicate a fluid volume deficit caused by internal or external bleeding. The nurse should examine the patients mouth, nose, and skin for bleeding. These vital signs do not indicate a pulmonary problem. Skin turgor and mucous membranes as well as urine output and level of consciousness may be assessed to determine the level of fluid deficit, but finding the source of blood loss is more important. Signs of gastrointestinal bleeding should also be assessed.

9. A patient who has recently had a myocardial infarction (MI) will begin taking clopidogrel (Plavix) to prevent a second MI. Which medication will the nurse expect the provider to order as adjunctive therapy for this patient? a. Aspirin b. Enoxaparin sodium (Lovenox) c. Ticagrelor (Brilinta) d. Warfarin (Coumadin)

ANS: A Aspirin is often used with clopidogrel to inhibit platelet aggregation to increase the effectiveness of this drug. Enoxaparin is used to prevent venous thrombosis. Ticagrelor is similar to clopidogrel and is not used along with clopidogrel. Warfarin is used to prevent thrombosis.

3. A client in sickle cell crisis is dehydrated and in the emergency department. The nurse plans to start an IV. Which fluid choice is best? a. 0.45% normal saline b. 0.9% normal saline c. Dextrose 50% (D50) d. Lactated Ringers solution

ANS: A Because clients in sickle cell crisis are often dehydrated, the fluid of choice is a hypotonic solution such as 0.45% normal saline. 0.9% normal saline and lactated Ringers solution are isotonic. D50 is hypertonic and not used for hydration.

4. A nursing student is caring for a client who had a myocardial infarction. The student is confused because the client states nothing is wrong and yet listens attentively while the student provides education on lifestyle changes and healthy menu choices. What response by the faculty member is best? a. Continue to educate the client on possible healthy changes. b. Emphasize complications that can occur with noncompliance. c. Tell the client that denial is normal and will soon go away. d. You need to make sure the client understands this illness.

ANS: A Clients are often in denial after a coronary event. The client who seems to be in denial but is compliant with treatment may be using a healthy form of coping that allows time to process the event and start to use problemfocused coping. The student should not discourage this type of denial and coping, but rather continue providing education in a positive manner. Emphasizing complications may make the client defensive and more anxious. Telling the client that denial is normal is placing too much attention on the process. Forcing the client to verbalize understanding of the illness is also potentially threatening to the client.

26. A client had an inferior wall myocardial infarction (MI). The nurse notes the clients cardiac rhythm as shown below: What action by the nurse is most important? a. Assess the clients blood pressure and level of consciousness. b. Call the health care provider or the Rapid Response Team. c. Obtain a permit for an emergency temporary pacemaker insertion. d. Prepare to administer antidysrhythmic medication.

ANS: A Clients with an inferior wall MI often have bradycardia and blocks that lead to decreased perfusion, as seen in this ECG strip showing sinus bradycardia. The nurse should first assess the clients hemodynamic status, including vital signs and level of consciousness. The client may or may not need the Rapid Response Team, a temporary pacemaker, or medication; there is no indication of this in the question.

2. A client hospitalized with sickle cell crisis frequently asks for opioid pain medications, often shortly after receiving a dose. The nurses on the unit believe the client is drug seeking. When the client requests pain medication, what action by the nurse is best? a. Give the client pain medication if it is time for another dose. b. Instruct the client not to request pain medication too early. c. Request the provider leave a prescription for a placebo. d. Tell the client it is too early to have more pain medication.

ANS: A Clients with sickle cell crisis often have severe pain that is managed with up to 48 hours of IV opioid analgesics. Even if the client is addicted and drug seeking, he or she is still in extreme pain. If the client can receive another dose of medication, the nurse should provide it. The other options are judgmental and do not address the clients pain. Giving placebos is unethical.

3. The health care provider tells the nurse that a client is to be started on a platelet inhibitor. About what drug does the nurse plan to teach the client? a. Clopidogrel (Plavix) b. Enoxaparin (Lovenox) c. Reteplase (Retavase) d. Warfarin (Coumadin)

ANS: A Clopidogrel is a platelet inhibitor. Enoxaparin is an indirect thrombin inhibitor. Reteplase is a fibrinolytic agent. Warfarin is a vitamin K antagonist.

9. A client is in the clinic a month after having a myocardial infarction. The client reports sleeping well since moving into the guest bedroom. What response by the nurse is best? a. Do you have any concerns about sexuality? b. I'm glad to hear you are sleeping well now. c. Sleep near your spouse in case of emergency. d. Why would you move into the guest room?

ANS: A Concerns about resuming sexual activity are common after cardiac events. The nurse should gently inquire if this is the issue. While it is good that the client is sleeping well, the nurse should investigate the reason for the move. The other two responses are likely to cause the client to be defensive.

21. A client is on a dopamine infusion via a peripheral line. What action by the nurse takes priority for safety? a. Assess the IV site hourly. b. Monitor the pedal pulses. c. Monitor the clients vital signs. d. Obtain consent for a central line.

ANS: A Dopamine should be infused through a central line to prevent extravasation and necrosis of tissue. If it needs to be run peripherally, the nurse assesses the site hourly for problems. When the client is getting the central line, ensuring informed consent is on the chart is a priority. But at this point, the client has only a peripheral line, so caution must be taken to preserve the integrity of the clients integumentary system. Monitoring pedal pulses and vital signs give indications as to how well the drug is working.

4. A patient who has received heparin after previous surgeries will be given enoxaparin sodium (Lovenox) after knee-replacement surgery. The patient asks how this drug is different from heparin. The nurse will explain that enoxaparin a. decreases the need for laboratory tests. b. has a shorter half-life than heparin. c. increases the risk of hemorrhage. d. may be taken orally instead of subcutaneously

ANS: A Enoxaparin is a lowmolecular-weight heparin, which produces more stable responses at lower doses, thus reducing the need for frequent lab monitoring. It has a longer half-life than heparin. It decreases the risk of hemorrhage because it is more stable at lower doses. It is given subcutaneously.

12. The nurse is caring for a client with leukemia who has the priority problem of fatigue. What action by the client best indicates that an important goal for this problem has been met? a. Doing activities of daily living (ADLs) using rest periods b. Helping plan a daily activity schedule c. Requesting a sleeping pill at night d. Telling visitors to leave when fatigued

ANS: A Fatigue is a common problem for clients with leukemia. This client is managing his or her own ADLs using rest periods, which indicates an understanding of fatigue and how to control it. Helping to plan an activity schedule is a lesser indicator. Requesting a sleeping pill does not help control fatigue during the day. Asking visitors to leave when tired is another lesser indicator. Managing ADLs using rest periods demonstrates the most comprehensive management strategy.

6. A client has Crohns disease. What type of anemia is this client most at risk for developing? a. Folic acid deficiency b. Fanconis anemia c. Hemolytic anemia d. Vitamin B12 anemia

ANS: A Malabsorption syndromes such as Crohns disease leave a client prone to folic acid deficiency. Fanconis anemia, hemolytic anemia, and vitamin B12 anemia are not related to Crohns disease.

12. A nurse assesses a client who is recovering from a myocardial infarction. The clients pulmonary artery pressure reading is 25/12 mm Hg. Which action should the nurse take first? a. Compare the results with previous pulmonary artery pressure readings. b. Increase the intravenous fluid rate because these readings are low. c. Immediately notify the health care provider of the elevated pressures. d. Document the finding in the clients chart as the only action.

ANS: A Normal pulmonary artery pressures range from 15 to 26 mm Hg for systolic and from 5 to 15 mm Hg for diastolic. Although this clients readings are within normal limits, the nurse needs to assess any trends that may indicate a need for medical treatment to prevent complications. There is no need to increase intravenous fluids or notify the provider.

11. An older adult is on cardiac monitoring after a myocardial infarction. The client shows frequent dysrhythmias. What action by the nurse is most appropriate? a. Assess for any hemodynamic effects of the rhythm. b. Prepare to administer antidysrhythmic medication. c. Notify the provider or call the Rapid Response Team. d. Turn the alarms off on the cardiac monitor.

ANS: A Older clients may have dysrhythmias due to age-related changes in the cardiac conduction system. They may have no significant hemodynamic effects from these changes. The nurse should first assess for the effects of the dysrhythmia before proceeding further. The alarms on a cardiac monitor should never be shut off. The other two actions may or may not be needed.

14. Which patient may require a higher than expected dose of an opioid analgesic? a. A patient with cancer b. A patient with a concussion c. A patient with hypotension d. A patient 3 days after surgery

ANS: A Opioids are titrated for oncology patients until pain relief is achieved or the side effects become intolerable, and extremely high doses may be required. Patient with closed head injuries should receive reduced doses of opioids if at all to reduce the risk of increased intracranial pressure. Patients with hypotension should receive reduced doses to prevent further decrease in blood pressure. Patients who are 3 days post-operation should not be experiencing severe pain.

2. A client is receiving rivaroxaban (Xarelto) and asks the nurse to explain how it works. What response by the nurse is best? a. It inhibits thrombin. b. It inhibits fibrinogen. c. It thins your blood. d. It works against vitamin K.

ANS: A Rivaroxaban is a direct thrombin inhibitor. It does not work on fibrinogen or vitamin K. It is not a blood thinner, although many clients call anticoagulants by this name.

19. A nurse cares for a client who is on a cardiac monitor. The monitor displayed the rhythm shown below: Which action should the nurse take first? a. Assess airway, breathing, and level of consciousness. b. Administer an amiodarone bolus followed by a drip. c. Cardiovert the client with a biphasic defibrillator. d. Begin cardiopulmonary resuscitation (CPR).

ANS: A Ventricular tachycardia occurs with repetitive firing of an irritable ventricular ectopic focus, usually at a rate of 140 to 180 beats/min or more. Ventricular tachycardia is a lethal dysrhythmia. The nurse should first assess if the client is alert and breathing. Then the nurse should call a Code Blue and begin CPR. If this client is pulseless, the treatment of choice is defibrillation. Amiodarone is the antidysrhythmic of choice, but it is not the first action.

17. A client has a platelet count of 9000/mm3. The nurse finds the client confused and mumbling. What action takes priority? a. Calling the Rapid Response Team b. Delegating taking a set of vital signs c. Instituting bleeding precautions d. Placing the client on bedrest

ANS: A With a platelet count this low, the client is at high risk of spontaneous bleeding. The most disastrous complication would be intracranial bleeding. The nurse needs to call the Rapid Response Team as this client has manifestations of a sudden neurologic change. The nurse should not delegate the vital signs as the client is no longer stable. Bleeding precautions will not address the immediate situation. Placing the client on bedrest or putting the client back into bed is important, but the critical action is to call for immediate medical attention.

8. A nurse is preparing to administer a blood transfusion to an older adult. Understanding age-related changes, what alterations in the usual protocol are necessary for the nurse to implement? (Select all that apply.) a. Assess vital signs more often. b. Hold other IV fluids running. c. Premedicate to prevent reactions. d. Transfuse smaller bags of blood. e. Transfuse each unit over 8 hours.

ANS: A, B The older adult needs vital signs monitored as often as every 15 minutes for the duration of the transfusion because changes may be the only indication of a transfusion-related problem. To prevent fluid overload, the nurse obtains a prescription to hold other running IV fluids during the transfusion. The other options are not warranted.

1. A nurse is caring for a client with a history of renal insufficiency who is scheduled for a cardiac catheterization. Which actions should the nurse take prior to the catheterization? (Select all that apply.) a. Assess for allergies to iodine. b. Administer intravenous fluids. c. Assess blood urea nitrogen (BUN) and creatinine results. d. Insert a Foley catheter. e. Administer a prophylactic antibiotic. f. Insert a central venous catheter.

ANS: A, B, C If the client has kidney disease (as indicated by BUN and creatinine results), fluids and Mucomyst may be given 12 to 24 hours before the procedure for renal protection. The client should be assessed for allergies to iodine, including shellfish; the contrast medium used during the catheterization contains iodine. A Foley catheter and central venous catheter are not required for the procedure and would only increase the clients risk for infection. Prophylactic antibiotics are not administered prior to a cardiac catheterization.

5. A nursing student planning to teach clients about risk factors for coronary artery disease (CAD) would include which topics? (Select all that apply.) a. Advanced age b. Diabetes c. Ethnic background d. Medication use e. Smoking

ANS: A, B, C, E Age, diabetes, ethnic background, and smoking are all risk factors for developing CAD; medication use is not.

9. A client has heparin-induced thrombocytopenia (HIT). The student nurse asks how this is treated. About what drugs does the nurse instructor teach? (Select all that apply.) a. Argatroban (Argatroban) b. Bivalirudin (Angiomax) c. Clopidogrel (Plavix) d. Lepirudin (Refludan) e. Methylprednisolone (Solu-Medrol)

ANS: A, B, D The standard drugs used to treat HIT are argatroban, bivalirudin, and lepirudin. The other drugs are not used. Clopidogrel is an antiplatelet agent used to reduce the likelihood of stroke or myocardial infarction. Methylprednisolone is a steroid used to reduce inflammation.

5. A student nurse is helping a registered nurse with a blood transfusion. Which actions by the student are most appropriate? (Select all that apply.) a. Hanging the blood product using normal saline and a filtered tubing set b. Taking a full set of vital signs prior to starting the blood transfusion c. Telling the client someone will remain at the bedside for the first 5 minutes d. Using gloves to start the clients IV if needed and to handle the blood product e. Verifying the clients identity, and checking blood compatibility and expiration time

ANS: A, B, D Correct actions prior to beginning a blood transfusion include hanging the product with saline and the correct filtered blood tubing, taking a full set of vital signs prior to starting, and using gloves. Someone stays with the client for the first 15 to 30 minutes of the transfusion. Two registered nurses must verify the clients identity and blood compatibility.

3. A nursing student studying acute coronary syndromes learns that the pain of a myocardial infarction (MI) differs from stable angina in what ways? (Select all that apply.) a. Accompanied by shortness of breath b. Feelings of fear or anxiety c. Lasts less than 15 minutes d. No relief from taking nitroglycerin e. Pain occurs without known cause

ANS: A, B, D, E The pain from an MI is often accompanied by shortness of breath and fear or anxiety. It lasts longer than 15 minutes and is not relieved by nitroglycerin. It occurs without a known cause such as exertion.

1. A student nurse learns that the spleen has several functions. What functions do they include? (Select all that apply.) a. Breaks down hemoglobin b. Destroys old or defective red blood cells (RBCs) c. Forms vitamin K for clotting d. Stores extra iron in ferritin e. Stores platelets not circulating

ANS: A, B, E Functions of the spleen include breaking down hemoglobin released from RBCs, destroying old or defective RBCs, and storing the platelets that are not in circulation. Forming vitamin K for clotting and storing extra iron in ferritin are functions of the liver.

1.The nurse is administering ibuprofen (Advil) to an older patient. Which assessment data causes the nurse to hold the medication? (Select all that apply.) a. Patient states allergy to aspirin. b. Patient states joint pain is 2/10 and intermittent. c. Patient reports past medical history of gastric ulcer. d. Patient reports last bowel movement was 4 days ago. e. Patient experiences respiratory depression after administration of an opioid medication

ANS: A, C NSAIDs can cause bleeding, especially in the gastrointestinal (GI) tract; therefore, NSAIDs are most likely contraindicated in this patient. Patients with an allergy to aspirin or have asthma are sometimes also allergic to other NSAIDs. The nurse needs to verify that the health care provider is aware of the history of GI bleeding and of allergy to aspirin before administering ibuprofen. NSAIDs do not interfere with bowel function and are used for the treatment of mild to moderate acute intermittent pain. NSAIDs also do not suppress the central nervous system by causing respiratory depression.

2. An older client asks the nurse why people my age have weaker immune systems than younger people. What responses by the nurse are best? (Select all that apply.) a. Bone marrow produces fewer blood cells. b. You may have decreased levels of circulating platelets. c. You have lower levels of plasma proteins in the blood. d. Lymphocytes become more reactive to antigens. e. Spleen function declines after age 60.

ANS: A, C The aging adult has bone marrow that produces fewer cells and decreased blood volume with fewer plasma proteins. Platelet numbers remain unchanged, lymphocytes become less reactive, and spleen function stays the same.

1. A nurse working with clients with sickle cell disease (SCD) teaches about self-management to prevent exacerbations and sickle cell crises. What factors should clients be taught to avoid? (Select all that apply.) a. Dehydration b. Exercise c. Extreme stress d. High altitudes e. Pregnancy

ANS: A, C, D, E Several factors cause red blood cells to sickle in SCD, including dehydration, extreme stress, high altitudes, and pregnancy. Strenuous exercise can also cause sickling, but not unless it is very vigorous

4. A nurse reviews a clients laboratory results. Which findings should alert the nurse to the possibility of atherosclerosis? (Select all that apply.) a. Total cholesterol: 280 mg/dL b. High-density lipoprotein cholesterol: 50 mg/dL c. Triglycerides: 200 mg/dL d. Serum albumin: 4 g/dL e. Low-density lipoprotein cholesterol: 160 mg/dL

ANS: A, C, E A lipid panel is often used to screen for cardiovascular risk. Total cholesterol, triglycerides, and low-density lipoprotein cholesterol levels are all high, indicating higher risk for cardiovascular disease. High-density lipoprotein cholesterol is within the normal range for both males and females. Serum albumin is not assessed for atherosclerosis.

2. A student studying leukemias learns the risk factors for developing this disorder. Which risk factors does this include? (Select all that apply.) a. Chemical exposure b. Genetically modified foods c. Ionizing radiation exposure d. Vaccinations e. Viral infections

ANS: A, C, E Chemical and ionizing radiation exposure and viral infections are known risk factors for developing leukemia. Eating genetically modified food and receiving vaccinations are not known risk factors.

2. A nurse is caring for a client who had coronary artery bypass grafting yesterday. What actions does the nurse delegate to the unlicensed assistive personnel (UAP)? (Select all that apply.) a. Assist the client to the chair for meals and to the bathroom. b. Encourage the client to use the spirometer every 4 hours. c. Ensure the client wears TED hose or sequential compression devices. d. Have the client rate pain on a 0-to-10 scale and report to the nurse. e. Take and record a full set of vital signs per hospital protocol.

ANS: A, C, E The nurse can delegate assisting the client to get up in the chair or ambulate to the bathroom, applying TEDs or sequential compression devices, and taking/recording vital signs. The spirometer should be used every hour the day after surgery. Assessing pain using a 0-to-10 scale is a nursing assessment, although if the client reports pain, the UAP should inform the nurse so a more detailed assessment is done.

6. A student nurse is learning about blood transfusion compatibilities. What information does this include? (Select all that apply.) a. Donor blood type A can donate to recipient blood type AB. b. Donor blood type B can donate to recipient blood type O. c. Donor blood type AB can donate to anyone. d. Donor blood type O can donate to anyone. e. Donor blood type A can donate to recipient blood type B.

ANS: A, D Blood type A can be donated to people who have blood types A or AB. Blood type O can be given to anyone. Blood type B can be donated to people who have blood types B or AB. Blood type AB can only go to recipients with blood type AB.

1. A nurse cares for a client with congestive heart failure who has a regular cardiac rhythm of 128 beats/min. For which physiologic alterations should the nurse assess? (Select all that apply.) a. Decrease in cardiac output b. Increase in cardiac output c. Decrease in blood pressure d. Increase in blood pressure e. Decrease in urine output f. Increase in urine output

ANS: A, D, E Elevated heart rates in a healthy client initially cause blood pressure and cardiac output to increase. However, in a client who has congestive heart failure or a client with long-term tachycardia, ventricular filling time, cardiac output, and blood pressure eventually decrease. As cardiac output and blood pressure decrease, urine output will fall.

6. A client is having a bone marrow biopsy today. What action by the nurse takes priority? a. Administer pain medication first. b. Ensure valid consent is on the chart. c. Have the client shower in the morning. d. Premedicate the client with sedatives

ANS: B A bone marrow biopsy is an invasive procedure that requires informed consent. Pain medication and sedation are important components of care for this client but do not take priority. The client may or may not need or be able to shower.

22. After assessing a client who is receiving an amiodarone intravenous infusion for unstable ventricular tachycardia, the nurse documents the findings and compares these with the previous assessment findings: Vital Signs Nursing Assessment Time: 0800 Temperature: 98 F Heart rate: 68 beats/min Blood pressure: 135/60 mm Hg Respiratory rate: 14 breaths/min Oxygen saturation: 96% Oxygen therapy: 2 L nasal cannula Time: 1000 Client alert and oriented. Cardiac rhythm: sinus bradycardia. Skin: warm, dry, and appropriate for race. Respirations equal and unlabored. Client denies shortness of breath and chest pain. Client voids 420 mL of clear yellow urine. Based on the assessments, which action should the nurse take? a. Stop the infusion and flush the IV. b. Slow the amiodarone infusion rate. c. Administer IV normal saline. d. Ask the client to cough and deep breathe.

ANS: B IV administration of amiodarone may cause bradycardia and atrioventricular (AV) block. The correct action for the nurse to take at this time is to slow the infusion, because the client is asymptomatic and no evidence reveals AV block that might require pacing. Abruptly ceasing the medication could allow fatal dysrhythmias to occur. The administration of IV fluids and encouragement of coughing and deep breathing exercises are not indicated, and will not increase the clients heart rate.

13. A nurse cares for a client who has an 80% blockage of the right coronary artery (RCA) and is scheduled for bypass surgery. Which intervention should the nurse be prepared to implement while this client waits for surgery? a. Administration of IV furosemide (Lasix) b. Initiation of an external pacemaker c. Assistance with endotracheal intubation d. Placement of central venous access

ANS: B The RCA supplies the right atrium, the right ventricle, the inferior portion of the left ventricle, and the atrioventricular (AV) node. It also supplies the sinoatrial node in 50% of people. If the client totally occludes the RCA, the AV node would not function and the client would go into heart block, so emergency pacing should be available for the client. Furosemide, intubation, and central venous access will not address the primary complication of RCA occlusion, which is AV node malfunction

20. A client receiving a blood transfusion develops anxiety and low back pain. After stopping the transfusion, what action by the nurse is most important? a. Documenting the events in the clients medical record b. Double-checking the client and blood product identification c. Placing the client on strict bedrest until the pain subsides d. Reviewing the clients medical record for known allergies

ANS: B This client had a hemolytic transfusion reaction, most commonly caused by blood type or Rh incompatibility. The nurse should double-check all identifying information for both the client and blood type. Documentation occurs after the client is stable. Bedrest may or may not be needed. Allergies to medications or environmental items is not related

8. A nurse cares for a client with an intravenous temporary pacemaker for bradycardia. The nurse observes the presence of a pacing spike but no QRS complex on the clients electrocardiogram. Which action should the nurse take next? a. Administer intravenous diltiazem (Cardizem). b. Assess vital signs and level of consciousness. c. Administer sublingual nitroglycerin. d. Assess capillary refill and temperature

ANS: B In temporary pacing, the wires are threaded onto the epicardial surface of the heart and exit through the chest wall. The pacemaker spike should be followed immediately by a QRS complex. Pacing spikes seen without subsequent QRS complexes imply loss of capture. If there is no capture, then there is no ventricular depolarization and contraction. The nurse should assess for cardiac output via vital signs and level of consciousness. The other interventions would not determine if the client is tolerating the loss of capture

9.A nurse is teaching a patient about patient-controlled analgesia (PCA). Which statement made by the patient indicates to the nurse that teaching is effective? a. "I will only need to be on this pain medication." b. "I feel less anxiety about the possibility of overdosing." c. "I can receive the pain medication as frequently as I need to." d. "I need the nurse to notify me when it is time for another dose."

ANS: B A PCA is a device that allows the patient to determine the level of pain relief delivered, reducing the risk of overdose. The PCA infusion pumps are designed to deliver a specific dose that is programmed to be available at specific time intervals (usually in the range of 8 to 15 minutes) when the patient activates the delivery button. A limit on the number of doses per hour or 4-hour interval may also be set. This can help decrease a patient's anxiety related to possible overdose. Its use also often eases anxiety because the patient is not reliant on the nurse for pain relief. Other medications, such as oral analgesics, can be given in addition to the PCA machine. One benefit of PCA is that the patient does not need to rely on the nurse to administer pain medication; the patient determines when to take the medication.

16.A patient who had a motor vehicle crash 2 days ago is experiencing pain and is receiving patient-controlled analgesia (PCA). Which assessment finding indicates effective pain management with the PCA? a. The patient is sleeping and is difficult to arouse. b. The patient rates pain at a level of 2 on a 0 to 10 scale. c. The patient has sufficient medication left in the PCA syringe. d. The patient presses the control button to deliver pain medication

ANS: B A level of 2 on a scale of 0 to 10 is evidence of effective pain management. The effectiveness of pain-relief measures is determined by the patient. If the patient is satisfied with the amount of pain relief, then pain measures are effective. A patient who is sleeping and is difficult to arouse is possibly oversedated; the nurse needs to assess this patient further. The amount of medication left in the PCA syringe does not indicate whether pain management is effective or not. Pressing the button shows that the patient knows how to use the PCA but does not evaluate pain management

7. A client has intra-arterial blood pressure monitoring after a myocardial infarction. The nurse notes the clients heart rate has increased from 88 to 110 beats/min, and the blood pressure dropped from 120/82 to 100/60 mm Hg. What action by the nurse is most appropriate? a. Allow the client to rest quietly. b. Assess the client for bleeding. c. Document the findings in the chart. d. Medicate the client for pain.

ANS: B A major complication related to intra-arterial blood pressure monitoring is hemorrhage from the insertion site. Since these vital signs are out of the normal range, are a change, and are consistent with blood loss, the nurse should assess the client for any bleeding associated with the arterial line. The nurse should document the findings after a full assessment. The client may or may not need pain medication and rest; the nurse first needs to rule out any emergent bleeding.

24. A nurse assesses a client who has mitral valve regurgitation. For which cardiac dysrhythmia should the nurse assess? a. Preventricular contractions b. Atrial fibrillation c. Symptomatic bradycardia d. Sinus tachycardia

ANS: B Atrial fibrillation is a clinical manifestation of mitral valve regurgitation and stenosis. Preventricular contractions and bradycardia are not associated with valvular problems. These are usually identified in clients with electrolyte imbalances, myocardial infarction, and sinus node problems. Sinus tachycardia is a manifestation of aortic regurgitation due to a decrease in cardiac output.

5. A nurse evaluates prescriptions for a client with chronic atrial fibrillation. Which medication should the nurse expect to find on this clients medication administration record to prevent a common complication of this condition? a. Sotalol (Betapace) b. Warfarin (Coumadin) c. Atropine (Sal-Tropine) d. Lidocaine (Xylocaine)

ANS: B Atrial fibrillation puts clients at risk for developing emboli. Clients at risk for emboli are treated with anticoagulants, such as heparin, enoxaparin, or warfarin. Sotalol, atropine, and lidocaine are not appropriate for this complication.

24.A nurse is caring for a patient with chronic pain. Which statement by the nurse indicates an understanding of pain management? a. "This patient says the pain is a 5 but is not acting like it. I am not going to give any pain medication." b. "I need to reassess the patient's pain 1 hour after administering oral pain medication." c. "It wasn't time for the patient's medication, so when it was requested, I gave a placebo." d. "The patient is sleeping, so I pushed the PCA button."

ANS: B Be sure to evaluate after an appropriate period of time. For instance, oral medications usually peak in about 1 hour, whereas IVP medications peak in 15 to 30 minutes. Ask a patient if a medication alleviates the pain when it is peaking. Because oral medications usually peak in about an hour, you need to reassess the patient's pain within an hour of administration. Nurses must believe any patient report of pain, even if nonverbal communication is not consistent with pain ratings. The patient is the only person who should push the PCA button. Pushing the PCA when a patient is sleeping is dangerous and may lead to narcotic overdose or respiratory depression. Giving the patient a placebo and telling the patient it is medication is unethical.

12. A nurse assists with the cardioversion of a client experiencing acute atrial fibrillation. Which action should the nurse take prior to the initiation of cardioversion? a. Administer intravenous adenosine. b. Turn off oxygen therapy. c. Ensure a tongue blade is available. d. Position the client on the left side.

ANS: B For safety during cardioversion, the nurse should turn off any oxygen therapy to prevent fire. The other interventions are not appropriate for a cardioversion. The client should be placed in a supine position.

6. A client has hemodynamic monitoring after a myocardial infarction. What safety precaution does the nurse implement for this client? a. Document pulmonary artery wedge pressure (PAWP) readings and assess their trends. b. Ensure the balloon does not remain wedged. c. Keep the client on strict NPO status. d. Maintain the client in a semi-Fowlers position.

ANS: B If the balloon remains inflated, it can cause pulmonary infarction or rupture. The nurse should ensure the balloon remains deflated between PAWP readings. Documenting PAWP readings and assessing trends is an important nursing action related to hemodynamic monitoring, but is not specifically related to safety. The client does not have to be NPO while undergoing hemodynamic monitoring. Positioning may or may not affect readings.

19. The nurse is caring for a client with a chest tube after a coronary artery bypass graft. The drainage slows significantly. What action by the nurse is most important? a. Increase the setting on the suction. b. Notify the provider immediately. c. Re-position the chest tube. d. Take the tubing apart to assess for clots.

ANS: B If the drainage in the chest tube decreases significantly and dramatically, the tube may be blocked by a clot. This could lead to cardiac tamponade. The nurse should notify the provider immediately. The nurse should not independently increase the suction, re-position the chest tube, or take the tubing apart.

18. A nurse is preparing to administer a blood transfusion. What action is most important? a. Correctly identifying client using two identifiers b. Ensuring informed consent is obtained if required c. Hanging the blood product with Ringers lactate d. Staying with the client for the entire transfusion

ANS: B If the facility requires informed consent for transfusions, this action is most important because it precedes the other actions taken during the transfusion. Correctly identifying the client and blood product is a National Patient Safety Goal, and is the most important action after obtaining informed consent. Ringers lactate is not used to transfuse blood. The nurse does not need to stay with the client for the duration of the transfusion

15. The nurse assesses an older patient 60 minutes after administering 4 mg of intravenous morphine sulfate (MS) for postoperative pain. The patients analgesia order is for 2 to 5 mg of MS IV every 2 hours. The nurse notes that the patient is lying very still. The patients heart rate is 96 beats per minute, respiratory rate is 14 breaths per minute, and blood pressure is 140/90 mm Hg. When asked to rate the level of pain, the patient replies just a 5. The nurse will perform which action? a. Give 3 mg of MS at the next dose. b. Give 5 mg of MS at the next dose. c. Request an order for an oral opioid to give now. d. Request an order for acetaminophen to give now.

ANS: B Older patients often minimize pain when asked, so the nurse should evaluate nonverbal cues to pain such as elevated heart rate and blood pressure and the fact that the patient is lying very still. The nurse should increase the dose the next time the pain medication is given

28.The nurse is assessing a patient for opioid tolerance. Which finding supports the nurse's assessment? a. The patient needed a substantial dose of naloxone (Narcan). b. The patient needs increasingly higher doses of opioid to control pain. c. The patient no longer experiences sedation from the usual dose of opioid. d. The patient asks for pain medication close to the time it is due around the clock

ANS: B Opioid tolerance occurs when a patient needs higher doses of an opioid to control pain. Naloxone (Narcan) is an opioid antagonist that is given to reverse the effects of opioid overdose. Taking pain medications regularly around the clock is an effective way to control pain. The pain medication for this patient is most likely effectively managing the patient's pain because the patient is not asking for the medication before it is due. A patient no longer experiencing a side effect (sedation) of an opioid does not indicate opioid tolerance.

33.A nurse is caring for a patient who fell on the ice and has connective tissue damage in the wrist and hand. The patient describes the pain as throbbing. Which type of pain does the nurse document in this patient's medical record? a. Visceral pain b. Somatic pain c. Centrally generated pain d. Peripherally generated pain

ANS: B Somatic pain comes from bone, joint, connective tissue, or muscle. Visceral pain arises from the visceral (internal) organs such as the GI tract and pancreas. Peripherally generated pain in the peripheral nerves can be caused by polyneuropathies or mononeuropathies. Centrally generated pain results from injury to the central or peripheral nervous system, causing deafferentation or sympathetically maintained pain.

17.The nurse is caring for a patient to ease modifiable factors that contribute to pain. Which areas did the nurse focus on with this patient? a. Age and gender b. Anxiety and fear c. Culture and ethnicity d. Previous pain experiences and cognitive abilities

ANS: B Some examples of modifiable contributors to pain are anxiety and fear. The nurse can take measures to ease the patient's anxiety and fear related to pain. Age, gender, culture, ethnicity, cognitive abilities, and previous pain experience are all nonmodifiable factors that the nurse can help the patient to understand, but the nurse cannot alter them.

16. A client has presented to the emergency department with an acute myocardial infarction (MI). What action by the nurse is best to meet The Joint Commissions Core Measures outcomes? a. Obtain an electrocardiogram (ECG) now and in the morning. b. Give the client an aspirin. c. Notify the Rapid Response Team. d. Prepare to administer thrombolytics.

ANS: B The Joint Commissions Core Measures set for acute MI require that aspirin is administered when a client with MI presents to the emergency department or when an MI occurs in the hospital. A rapid ECG is vital, but getting another one in the morning is not part of the Core Measures set. The Rapid Response Team is not needed if an emergency department provider is available. Thrombolytics may or may not be needed

23. A client admitted for sickle cell crisis is distraught after learning her child also has the disease. What response by the nurse is best? a. Both you and the father are equally responsible for passing it on. b. I can see you are upset. I can stay here with you a while if you like. c. Its not your fault; there is no way to know who will have this disease. d. There are many good treatments for sickle cell disease these days.

ANS: B The best response is for the nurse to offer self, a therapeutic communication technique that uses presence. Attempting to assign blame to both parents will not help the client feel better. There is genetic testing available, so it is inaccurate to state there is no way to know who will have the disease. Stating that good treatments exist belittles the clients feelings.

10. After teaching a client who has an implantable cardioverter-defibrillator (ICD), a nurse assesses the clients understanding. Which statement by the client indicates a correct understanding of the teaching? a. I should wear a snug-fitting shirt over the ICD. b. I will avoid sources of strong electromagnetic fields. c. I should participate in a strenuous exercise program. d. Now I can discontinue my antidysrhythmic medication.

ANS: B The client being discharged with an ICD is instructed to avoid strong sources of electromagnetic fields. Clients should avoid tight clothing, which could cause irritation over the ICD generator. The client should be encouraged to exercise but should not engage in strenuous activities that cause the heart rate to meet or exceed the ICD cutoff point because the ICD can discharge inappropriately. The client should continue all prescribed medications.

16. A nurse assesses a client with pericarditis. Which assessment finding should the nurse expect to find? a. Heart rate that speeds up and slows down b. Friction rub at the left lower sternal border c. Presence of a regular gallop rhythm d. Coarse crackles in bilateral lung bases

ANS: B The client with pericarditis may present with a pericardial friction rub at the left lower sternal border. This sound is the result of friction from inflamed pericardial layers when they rub together. The other assessments are not related.

21. The nurse is caring for a client on the medical-surgical unit who suddenly becomes unresponsive and has no pulse. The cardiac monitor shows the rhythm below: After calling for assistance and a defibrillator, which action should the nurse take next? a. Perform a pericardial thump. b. Initiate cardiopulmonary resuscitation (CPR). c. Start an 18-gauge intravenous line. d. Ask the clients family about code status.

ANS: B The clients rhythm is ventricular fibrillation. This is a lethal rhythm that is best treated with immediate defibrillation. While the nurse is waiting for the defibrillator to arrive, the nurse should start CPR. A pericardial thump is not a treatment for ventricular fibrillation. If the client does not already have an IV, other members of the team can insert one after defibrillation. The clients code status should already be known by the nurse prior to this event.

11. A nurse cares for a client who is prescribed magnetic resonance imaging (MRI) of the heart. The clients health history includes a previous myocardial infarction and pacemaker implantation. Which action should the nurse take? a. Schedule an electrocardiogram just before the MRI. b. Notify the health care provider before scheduling the MRI. c. Call the physician and request a laboratory draw for cardiac enzymes. d. Instruct the client to increase fluid intake the day before the MRI.

ANS: B The magnetic fields of the MRI can deactivate the pacemaker. The nurse should call the health care provider and report that the client has a pacemaker so the provider can order other diagnostic tests. The client does not need an electrocardiogram, cardiac enzymes, or increased fluids.

5. A client has a serum ferritin level of 8 ng/mL and microcytic red blood cells. What action by the nurse is best? a. Encourage high-protein foods. b. Perform a Hemoccult test on the clients stools. c. Offer frequent oral care. d. Prepare to administer cobalamin (vitamin B12).

ANS: B This client has laboratory findings indicative of iron deficiency anemia. The most common cause of this disorder is blood loss, often from the GI tract. The nurse should perform a Hemoccult test on the clients stools. High-protein foods may help the condition, but dietary interventions take time to work. That still does not determine the cause. Frequent oral care is not related. Cobalamin injections are for pernicious anemia

3. A client is in the hospital after suffering a myocardial infarction and has bathroom privileges. The nurse assists the client to the bathroom and notes the clients O2 saturation to be 95%, pulse 88 beats/min, and respiratory rate 16 breaths/min after returning to bed. What action by the nurse is best? a. Administer oxygen at 2 L/min. b. Allow continued bathroom privileges. c. Obtain a bedside commode. d. Suggest the client use a bedpan.

ANS: B This clients physiologic parameters did not exceed normal during and after activity, so it is safe for the client to continue using the bathroom. There is no indication that the client needs oxygen, a commode, or a bedpan.

2. A nurse cares for a client who has a heart rate averaging 56 beats/min with no adverse symptoms. Which activity modification should the nurse suggest to avoid further slowing of the heart rate? a. Make certain that your bath water is warm. b. Avoid straining while having a bowel movement. c. Limit your intake of caffeinated drinks to one a day. d. Avoid strenuous exercise such as running.

ANS: B Bearing down strenuously during a bowel movement is one type of Valsalva maneuver, which stimulates the vagus nerve and results in slowing of the heart rate. Such a response is not desirable in a person who has bradycardia. The other instructions are not appropriate for this condition.

14. A nurse teaches a client with diabetes mellitus and a body mass index of 42 who is at high risk for coronary artery disease. Which statement related to nutrition should the nurse include in this clients teaching? a. The best way to lose weight is a high-protein, low-carbohydrate diet. b. You should balance weight loss with consuming necessary nutrients. c. A nutritionist will provide you with information about your new diet. d. If you exercise more frequently, you wont need to change your diet.

ANS: B Clients at risk for cardiovascular diseases should follow the American Heart Association guidelines to combat obesity and improve cardiac health. The nurse should encourage the client to eat vegetables, fruits, unrefined whole-grain products, and fat-free dairy products while losing weight. High-protein food items are often high in fat and calories. Although the nutritionist can assist with client education, the nurse should include nutrition education and assist the client to make healthy decisions. Exercising and eating nutrient-rich foods are both important components in reducing cardiovascular risk.

4. A nurse assesses a client with atrial fibrillation. Which manifestation should alert the nurse to the possibility of a serious complication from this condition? a. Sinus tachycardia b. Speech alterations c. Fatigue d. Dyspnea with activity

ANS: B Clients with atrial fibrillation are at risk for embolic stroke. Evidence of embolic events includes changes in mentation, speech, sensory function, and motor function. Clients with atrial fibrillation often have a rapid ventricular response as a result. Fatigue is a nonspecific complaint. Clients with atrial fibrillation often have dyspnea as a result of the decreased cardiac output caused by the rhythm disturbance.

7. A nurse in a hematology clinic is working with four clients who have polycythemia vera. Which client should the nurse see first? a. Client with a blood pressure of 180/98 mm Hg b. Client who reports shortness of breath c. Client who reports calf tenderness and swelling d. Client with a swollen and painful left great toe

ANS: B Clients with polycythemia vera often have clotting abnormalities due to the hyperviscous blood with sluggish flow. The client reporting shortness of breath may have a pulmonary embolism and should be seen first. The client with a swollen calf may have a deep vein thrombosis and should be seen next. High blood pressure and gout symptoms are common findings with this disorder.

20. A home health care nurse is visiting an older client who lives alone after being discharged from the hospital after a coronary artery bypass graft. What finding in the home most causes the nurse to consider additional referrals? a. Dirty carpets in need of vacuuming b. Expired food in the refrigerator c. Old medications in the kitchen d. Several cats present in the home

ANS: B Expired food in the refrigerator demonstrates a safety concern for the client and a possible lack of money to buy food. The nurse can consider a referral to Meals on Wheels or another home-based food program. Dirty carpets may indicate the client has no household help and is waiting for clearance to vacuum. Old medications can be managed by the home health care nurse and the client working collaboratively. Having pets is not a cause for concern.

18. The nurse is caring for a patient who is receiving clopidogrel (Plavix). The patient calls the nurse to report flulike symptoms. The nurse notes a heart rate of 76 beats per minute, a blood pressure of 110/76 mm Hg, and a respiratory rate of 20 breaths per minute. The nurse suspects that the patient is experiencing which condition? a. Anaphylaxis b. An expected drug side effect c. Hemorrhage d. Possible myocardial infarction

ANS: B Flulike symptoms are a side effect of clopidogrel. The patient has normal vital signs. Anaphylaxis is characterized by respiratory distress and hypotension. Hemorrhage is characterized by tachycardia and hypotension.

14. A nurse is in charge of the coronary intensive care unit. Which client should the nurse see first? a. Client on a nitroglycerin infusion at 5 mcg/min, not titrated in the last 4 hours b. Client who is 1 day post coronary artery bypass graft, blood pressure 180/100 mm Hg c. Client who is 1 day post percutaneous coronary intervention, going home this morning d. Client who is 2 days post coronary artery bypass graft, became dizzy this a.m. while walking

ANS: B Hypertension after coronary artery bypass graft surgery can be dangerous because it puts too much pressure on the suture lines and can cause bleeding. The charge nurse should see this client first. The client who became dizzy earlier should be seen next. The client on the nitroglycerin drip is stable. The client going home can wait until the other clients are cared for.

9. The family of a neutropenic client reports the client is not acting right. What action by the nurse is the priority? a. Ask the client about pain. b. Assess the client for infection. c. Delegate taking a set of vital signs. d. Look at todays laboratory results.

ANS: B Neutropenic clients often do not have classic manifestations of infection, but infection is the most common cause of death in neutropenic clients. The nurse should assess for infection. The nurse should assess for pain but this is not the priority. The nurse should take the clients vital signs instead of delegating them since the client has had a change in status. Laboratory results may be inconclusive.

15. A client with coronary artery disease (CAD) asks the nurse about taking fish oil supplements. What response by the nurse is best? a. Fish oil is contraindicated with most drugs for CAD. b. The best source is fish, but pills have benefits too. c. There is no evidence to support fish oil use with CAD. d. You can reverse CAD totally with diet and supplements.

ANS: B Omega-3 fatty acids have shown benefit in reducing lipid levels, in reducing the incidence of sudden cardiac death, and for stabilizing atherosclerotic plaque. The best source is fish three times a week or some fish oil supplements. The other options are not accurate.

7. A patient has been taking atorvastatin (Lipitor) for several months to treat hyperlipidemia. The patient reports muscle weakness and tenderness. The nurse will counsel the patient to a. ask the provider about switching to simvastatin. b. contact the provider to report these symptoms. c. start taking ibuprofen to combat these effects. d. stop taking the medication immediately.

ANS: B Patients taking statins should report immediately any muscle aches or weakness, which can lead to rhabdomyolysis, a muscle disintegration that can become fatal. All statins carry this risk, so changing to another statin is not indicated. Ibuprofen may be useful, but notifying the provider is essential. Patients should not abruptly discontinue statins without discussing this with the provider.

3. A patient has been receiving intravenous heparin. When laboratory tests are drawn, the nurse has difficulty stopping bleeding at the puncture site. The patient has bloody stools and is reporting abdominal pain.The nurse notes elevated partial thromboplastin time (PTT) and activated partial thromboplastin time (aPTT). Which action will the nurse perform? a. Ask for an order for oral warfarin (Coumadin). b. Obtain an order for protamine sulfate. c. Request an order for vitamin K. d. Suggest that the patient receive subcutaneous heparin

ANS: B Protamine sulfate is given as an antidote to heparin when patients clotting times are elevated. Oral warfarin will not stop the anticoagulant effects of heparin. Vitamin K is used as an antidote for warfarin. Administering heparin by another route is not indicated when there is a need to reverse the effects of heparin.

13. A nurse cares for a client recovering from prosthetic valve replacement surgery. The client asks, Why will I need to take anticoagulants for the rest of my life? How should the nurse respond? a. The prosthetic valve places you at greater risk for a heart attack. b. Blood clots form more easily in artificial replacement valves. c. The vein taken from your leg reduces circulation in the leg. d. The surgery left a lot of small clots in your heart and lungs.

ANS: B Synthetic valve prostheses and scar tissue provide surfaces on which platelets can aggregate easily and initiate the formation of blood clots. The other responses are inaccurate

3. A client has Hodgkins lymphoma, Ann Arbor stage Ib. For what manifestations should the nurse assess the client? (Select all that apply.) a. Headaches b. Night sweats c. Persistent fever d. Urinary frequency e. Weight loss

ANS: B, C, E In this stage, the disease is located in a single lymph node region or a single nonlymph node site. The client displays night sweats, persistent fever, and weight loss. Headache and urinary problems are not related.

4. A nurse works in a gerontology clinic. What age-related changes cause the nurse to alter standard assessment techniques from those used for younger adults? (Select all that apply.) a. Dentition deteriorates with more cavities. b. Nail beds may be thickened or discolored. c. Progressive loss of hair occurs with age. d. Sclerae begin to turn yellow or pale. e. Skin becomes dry as the client ages.

ANS: B, C, E Common findings in older adults include thickened or discolored nail beds, dry skin, and thinning hair. The nurse adapts to these changes by altering assessment techniques. Having more dental caries and changes in the sclerae are not normal age-related changes.

2. An emergency room nurse assesses a female client. Which assessment findings should alert the nurse to request a prescription for an electrocardiogram? (Select all that apply.) a. Hypertension b. Fatigue despite adequate rest c. Indigestion d. Abdominal pain e. Shortness of breath

ANS: B, C, E Women may not have chest pain with myocardial infarction, but may feel discomfort or indigestion. They often present with a triad of symptoms indigestion or feeling of abdominal fullness, feeling of chronic fatigue despite adequate rest, and feeling unable to catch their breath. Frequently, women are not diagnosed and therefore are not treated adequately. Hypertension and abdominal pain are not associated with acute coronary syndrome.

7. A client with chronic anemia has had many blood transfusions. What medications does the nurse anticipate teaching the client about adding to the regimen? (Select all that apply.) a. Azacitidine (Vidaza) b. Darbepoetin alfa (Aranesp) c. Decitabine (Dacogen) d. Epoetin alfa (Epogen) e. Methylprednisolone (Solu-Medrol)

ANS: B, D Darbepoetin alfa and epoetin alfa are both red blood cell colony-stimulating factors that will help increase the production of red blood cells. Azacitidine and decitabine are used for myelodysplastic syndromes. Methylprednisolone is a steroid and would not be used for this problem.

3. A nurse assesses a client who is recovering after a coronary catheterization. Which assessment findings in the first few hours after the procedure require immediate action by the nurse? (Select all that apply.) a. Blood pressure of 140/88 mm Hg b. Serum potassium of 2.9 mEq/L c. Warmth and redness at the site d. Expanding groin hematoma e. Rhythm changes on the cardiac monitor

ANS: B, D, E In the first few hours postprocedure, the nurse monitors for complications such as bleeding from the insertion site, hypotension, acute closure of the vessel, dye reaction, hypokalemia, and dysrhythmias. The clients blood pressure is slightly elevated but does not need immediate action. Warmth and redness at the site would indicate an infection, but this would not be present in the first few hours.

5. A nurse prepares a client for a pharmacologic stress echocardiogram. Which actions should the nurse take when preparing this client for the procedure? (Select all that apply.) a. Assist the provider to place a central venous access device. b. Prepare for continuous blood pressure and pulse monitoring. c. Administer the clients prescribed beta blocker. d. Give the client nothing by mouth 3 to 6 hours before the procedure. e. Explain to the client that dobutamine will simulate exercise for this examination.

ANS: B, D, E Clients receiving a pharmacologic stress echocardiogram will need peripheral venous access and continuous blood pressure and pulse monitoring. The client must be NPO 3 to 6 hours prior to the procedure. Education about dobutamine, which will be administered during the procedure, should be performed. Beta blockers are often held prior to the procedure

21. A client has thrombocytopenia. What client statement indicates the client understands self-management of this condition? a. I brush and use dental floss every day. b. I chew hard candy for my dry mouth. c. I usually put ice on bumps or bruises. d. Nonslip socks are best when I walk.

ANS: C The client should be taught to apply ice to areas of minor trauma. Flossing is not recommended. Hard foods should be avoided. The client should wear well-fitting shoes when ambulating

7. A nurse is caring for four clients. After reviewing todays laboratory results, which client should the nurse see first? a. Client with an international normalized ratio of 2.8 b. Client with a platelet count of 128,000/mm3 c. Client with a prothrombin time (PT) of 28 seconds d. Client with a red blood cell count of 5.1 million/L

ANS: C A normal PT is 11 to 12.5 seconds. This client is at high risk of bleeding. The other values are within normal limits.

24. The provider requests the nurse start an infusion of an inotropic agent on a client. How does the nurse explain the action of these drugs to the client and spouse? a. It constricts vessels, improving blood flow. b. It dilates vessels, which lessens the work of the heart. c. It increases the force of the hearts contractions. d. It slows the heart rate down for better filling.

ANS: C A positive inotrope is a medication that increases the strength of the hearts contractions. The other options are not correct.

20.The nurse has brought a patient the scheduled pain medication. The patient asks the nurse to wait to give pain medication until the time for the dressing change, which is 2 hours away. Which response by the nurse is most therapeutic? a. "This medication will still be providing you relief at the time of your dressing change." b. "OK, swallow this pain pill, and I will return in a minute to change your dressing." c. "Would you like medication to be given for dressing changes in addition to your regularly scheduled medication?" d. "Your medication is scheduled for this time, and I can't adjust the time for you. I'm sorry, but you must take your pill right now."

ANS: C Additional doses of medication can be given to patients in certain circumstances, as with an extensive dressing change, when the health care provider is notified that more medication is needed. It is the nurse's responsibility to communicate with the provider and with the patient about a pain-control plan that works for both. By asking to hold off on the dose, the patient is indicating that the dressing changes are extremely painful. The regularly scheduled dose might not be as effective for the patient 2 hours later when the dressing change is scheduled. Oral medications take 30 to 60 minutes to take effect. If the nurse began the dressing change right then, the medication would not have been absorbed yet. The patient has the right to refuse to take a medication.

13. A nurse is caring for a young male client with lymphoma who is to begin treatment. What teaching topic is a priority? a. Genetic testing b. Infection prevention c. Sperm banking d. Treatment options

ANS: C All teaching topics are important to the client with lymphoma, but for a young male, sperm banking is of particular concern if the client is going to have radiation to the lower abdomen or pelvis.

13. A client has an intra-arterial blood pressure monitoring line. The nurse notes bright red blood on the clients sheets. What action should the nurse perform first? a. Assess the insertion site. b. Change the clients sheets. c. Put on a pair of gloves. d. Assess blood pressure.

ANS: C For the nurses safety, he or she should put on a pair of gloves to prevent blood exposure. The other actions are appropriate as well, but first the nurse must don a pair of gloves.

7. A nurse assesses an older adult client who is experiencing a myocardial infarction. Which clinical manifestation should the nurse expect? a. Excruciating pain on inspiration b. Left lateral chest wall pain c. Disorientation and confusion d. Numbness and tingling of the arm

ANS: C In older adults, disorientation or confusion may be the major manifestation of myocardial infarction caused by poor cardiac output. Pain manifestations and numbness and tingling of the arm could also be related to the myocardial infarction. However, the nurse should be more concerned about the new onset of disorientation or confusion caused by decreased perfusion.

22. A client had an acute myocardial infarction. What assessment finding indicates to the nurse that a significant complication has occurred? a. Blood pressure that is 20 mm Hg below baseline b. Oxygen saturation of 94% on room air c. Poor peripheral pulses and cool skin d. Urine output of 1.2 mL/kg/hr for 4 hours

ANS: C Poor peripheral pulses and cool skin may be signs of impending cardiogenic shock and should be reported immediately. A blood pressure drop of 20 mm Hg is not worrisome. An oxygen saturation of 94% is just slightly below normal. A urine output of 1.2 mL/kg/hr for 4 hours is normal.

23. A client presents to the emergency department with an acute myocardial infarction (MI) at 1500 (3:00 PM). The facility has 24-hour catheterization laboratory abilities. To meet The Joint Commissions Core Measures set, by what time should the client have a percutaneous coronary intervention performed? a. 1530 (3:30 PM) b. 1600 (4:00 PM) c. 1630 (4:30 PM) d. 1700 (5:00 PM)

ANS: C The Joint Commissions Core Measures set for MI includes percutaneous coronary intervention within 90 minutes of diagnosis of myocardial infarction. Therefore, the client should have a percutaneous coronary intervention performed no later than 1630 (4:30 PM).

3. A nurse assesses clients on a medical-surgical unit. Which client should the nurse identify as having the greatest risk for cardiovascular disease? a. An 86-year-old man with a history of asthma b. A 32-year-old Asian-American man with colorectal cancer c. A 45-year-old American Indian woman with diabetes mellitus d. A 53-year-old postmenopausal woman who is on hormone therapy

ANS: C The incidence of coronary artery disease and hypertension is higher in American Indians than in whites or Asian Americans. Diabetes mellitus increases the risk for hypertension and coronary artery disease in people of any race or ethnicity. Asthma, colorectal cancer, and hormone therapy do not increase risk for cardiovascular disease.

17. A nurse prepares a client for coronary artery bypass graft surgery. The client states, I am afraid I might die. How should the nurse respond? a. This is a routine surgery and the risk of death is very low. b. Would you like to speak with a chaplain prior to surgery? c. Tell me more about your concerns about the surgery. d. What support systems do you have to assist you?

ANS: C The nurse should discuss the clients feelings and concerns related to the surgery. The nurse should not provide false hope or push the clients concerns off on the chaplain. The nurse should address support systems after addressing the clients current issue.

27. A client has been treated for a deep vein thrombus and today presents to the clinic with petechiae. Laboratory results show a platelet count of 42,000/mm3. The nurse reviews the clients medication list to determine if the client is taking which drug? a. Enoxaparin (Lovenox) b. Salicylates (aspirin) c. Unfractionated heparin d. Warfarin (Coumadin)

ANS: C This client has manifestations of heparin-induced thrombocytopenia. Enoxaparin, salicylates, and warfarin do not cause this condition.

20. A nurse performs an admission assessment on a 75-year-old client with multiple chronic diseases. The clients blood pressure is 135/75 mm Hg and oxygen saturation is 94% on 2 liters per nasal cannula. The nurse assesses the clients rhythm on the cardiac monitor and observes the reading shown below: Which action should the nurse take first? a. Begin external temporary pacing. b. Assess peripheral pulse strength. c. Ask the client what medications he or she takes. d. Administer 1 mg of atropine.

ANS: C This client is stable and therefore does not require any intervention except to determine the cause of the bradycardia. Bradycardia is often caused by medications. Clients who have multiple chronic diseases are often on multiple medications that can interact with each other. The nurse should assess the clients current medications first.

7. A telemetry nurse assesses a client with third-degree heart block who has wide QRS complexes and a heart rate of 35 beats/min on the cardiac monitor. Which assessment should the nurse complete next? a. Pulmonary auscultation b. Pulse strength and amplitude c. Level of consciousness d. Mobility and gait stability

ANS: C A heart rate of 40 beats/min or less with widened QRS complexes could have hemodynamic consequences. The client is at risk for inadequate cerebral perfusion. The nurse should assess for level of consciousness, lightheadedness, confusion, syncope, and seizure activity. Although the other assessments should be completed, the clients level of consciousness is the priority.

9. A nurse assesses a client who is recovering after a left-sided cardiac catheterization. Which assessment finding requires immediate intervention? a. Urinary output less than intake b. Bruising at the insertion site c. Slurred speech and confusion d. Discomfort in the left leg

ANS: C A left-sided cardiac catheterization specifically increases the risk for a cerebral vascular accident. A change in neurologic status needs to be acted on immediately. Discomfort and bruising are expected at the site. If intake decreases, a client can become dehydrated because of dye excretion. The second intervention would be to increase the clients fluid status. Neurologic changes would take priority

10. A client in the cardiac stepdown unit reports severe, crushing chest pain accompanied by nausea and vomiting. What action by the nurse takes priority? a. Administer an aspirin. b. Call for an electrocardiogram (ECG). c. Maintain airway patency. d. Notify the provider.

ANS: C Airway always is the priority. The other actions are important in this situation as well, but the nurse should stay with the client and ensure the airway remains patent (especially if vomiting occurs) while another person calls the provider (or Rapid Response Team) and facilitates getting an ECG done. Aspirin will probably be administered, depending on the providers prescription and the clients current medications.

17. After teaching a client who is recovering from a heart transplant to change positions slowly, the client asks, Why is this important? How should the nurse respond? a. Rapid position changes can create shear and friction forces, which can tear out your internal vascular sutures. b. Your new vascular connections are more sensitive to position changes, leading to increased intravascular pressure and dizziness. c. Your new heart is not connected to the nervous system and is unable to respond to decreases in blood pressure caused by position changes. d. While your heart is recovering, blood flow is diverted away from the brain, increasing the risk for stroke

ANS: C Because the new heart is denervated, the baroreceptor and other mechanisms that compensate for blood pressure drops caused by position changes do not function. This allows orthostatic hypotension to persist in the postoperative period. The other options are false statements and do not correctly address the clients question

16. A nurse cares for a client who is recovering from a myocardial infarction. The client states, I will need to stop eating so much chili to keep that indigestion pain from returning. How should the nurse respond? a. Chili is high in fat and calories; it would be a good idea to stop eating it. b. The provider has prescribed an antacid for you to take every morning. c. What do you understand about what happened to you? d. When did you start experiencing this indigestion?

ANS: C Clients who experience myocardial infarction often respond with denial, which is a defense mechanism. The nurse should ask the client what he or she thinks happened, or what the illness means to him or her. The other responses do not address the clients misconception about recent pain and the cause of that pain.

11. A nurse cares for a client with atrial fibrillation who reports fatigue when completing activities of daily living. What interventions should the nurse implement to address this clients concerns? a. Administer oxygen therapy at 2 liters per nasal cannula. b. Provide the client with a sleeping pill to stimulate rest. c. Schedule periods of exercise and rest during the day. d. Ask unlicensed assistive personnel to help bathe the client

ANS: C Clients who have atrial fibrillation are at risk for decreased cardiac output and fatigue when completing activities of daily living. The nurse should schedule periods of exercise and rest during the day to decrease fatigue. The other interventions will not assist the client with self-care activities

8. A nurse assesses a client 2 hours after a cardiac angiography via the left femoral artery. The nurse notes that the left pedal pulse is weak. Which action should the nurse take? a. Elevate the leg and apply a sandbag to the entrance site. b. Increase the flow rate of intravenous fluids. c. Assess the color and temperature of the left leg. d. Document the finding as left pedal pulse of +1/4.

ANS: C Loss of a pulse distal to an angiography entry site is serious, indicating a possible arterial obstruction. The pulse may be faint because of edema. The left pulse should be compared with the right, and pulses should be compared with previous assessments, especially before the procedure. Assessing color (pale, cyanosis) and temperature (cool, cold) will identify a decrease in circulation. Once all peripheral and vascular assessment data are acquired, the primary health care provider should be notified. Simply documenting the findings is inappropriate. The leg should be positioned below the level of the heart or dangling to increase blood flow to the distal portion of the leg. Increasing intravenous fluids will not address the clients problem

4. A nurse assesses an older adult client who has multiple chronic diseases. The clients heart rate is 48 beats/min. Which action should the nurse take first? a. Document the finding in the chart. b. Initiate external pacing. c. Assess the clients medications. d. Administer 1 mg of atropine.

ANS: C Pacemaker cells in the conduction system decrease in number as a person ages, resulting in bradycardia. The nurse should check the medication reconciliation for medications that might cause such a drop in heart rate, then should inform the health care provider. Documentation is important, but it is not the priority action. The heart rate is not low enough for atropine or an external pacemaker to be needed.

15. The nurse is assessing a patient prior to administering thrombolytic therapy. Which is an important assessment for this patient? a. Determining whether the patient has a history of diabetes b. Finding out about a history of renal disease c. Assessing which medications are taken for discomfort d. Assessing whether the patient eats green, leafy vegetables

ANS: C Patients who take aspirin or nonsteroidal antiinflammatory drugs (NSAIDs) should be monitored closely for excessive bleeding when given thrombolytics. There are no contraindications or precautions for patients with diabetes or renal disease. Foods rich in vitamin K are of concern for patients taking warfarin.

10. A nurse is caring for a client who is about to receive a bone marrow transplant. To best help the client cope with the long recovery period, what action by the nurse is best? a. Arrange a visitation schedule among friends and family. b. Explain that this process is difficult but must be endured. c. Help the client find things to hope for each day of recovery. d. Provide plenty of diversionary activities for this time.

ANS: C Providing hope is an essential nursing function during treatment for any disease process, but especially during the recovery period after bone marrow transplantation, which can take up to 3 weeks. The nurse can help the client look ahead to the recovery period and identify things to hope for during this time. Visitors are important to clients, but may pose an infection risk. Telling the client the recovery period must be endured does not acknowledge his or her feelings. Diversionary activities are important, but not as important as instilling hope.

8. The nurse provides teaching to a patient who will begin taking simvastatin (Zocor) to treat hyperlipidemia. Which statement by the patient indicates understanding of the teaching? a. I may have diarrhea as a result of taking this medication. b. I may stop taking this medication when my lipid levels are normal. c. I will need an annual eye examination while taking this medication. d. I will increase my intake of vitamins A, D, and E while taking this medication.

ANS: C The statins can affect visual acuity, so patients should be counseled to have annual eye examinations for assessment of cataract formation. The bile acid sequestrants, not statins, cause diarrhea. Statin drug therapy is lifelong or until behavioral changes prove equally effective (uncommon). Bile acid sequestrants, not statins, decrease the absorption of fat-soluble vitamins.

6. A patient who is taking warfarin has an international normalized ratio (INR) of 5.5. The nurse will anticipate giving a. fresh frozen plasma. b. intravenous iron. c. oral vitamin K. d. protamine sulfate.

ANS: C Vitamin K is an antagonist against warfarin, an oral anticoagulant. Patients with an INR of 5.5 should be given a low dose of oral vitamin K. Too much vitamin K may reduce the effectiveness of warfarin for up to 2 weeks. Fresh frozen plasma and intravenous iron are given for anemia caused by blood loss. Protamine sulfate is given for heparin overdose.

10. A client has received a bone marrow transplant and is waiting for engraftment. What actions by the nurse are most appropriate? (Select all that apply.) a. Not allowing any visitors until engraftment b. Limiting the protein in the clients diet c. Placing the client in protective precautions d. Teaching visitors appropriate hand hygiene e. Telling visitors not to bring live flowers or plants

ANS: C, D, E The client waiting for engraftment after bone marrow transplant has no white cells to protect him or her against infection. The client is on protective precautions and visitors are taught hand hygiene. No fresh flowers or plants are allowed due to the standing water in the vase or container that may harbor organisms. Limiting protein is not a healthy option and will not promote engraftment

24. A client with sickle cell disease (SCD) takes hydroxyurea (Droxia). The client presents to the clinic reporting an increase in fatigue. What laboratory result should the nurse report immediately? a. Hematocrit: 25% b. Hemoglobin: 9.2 mg/dL c. Potassium: 3.2 mEq/L d. White blood cell count: 38,000/mm3

ANS: D Although individuals with SCD often have elevated white blood cell (WBC) counts, this extreme elevation could indicate leukemia, a complication of taking hydroxyurea. The nurse should report this finding immediately. Alternatively, it could indicate infection, a serious problem for clients with SCD. Hematocrit and hemoglobin levels are normally low in people with SCD. The potassium level, while slightly low, is not as worrisome as the WBCs.

10. A nurse assesses a client who is scheduled for a cardiac catheterization. Which assessment should the nurse complete prior to this procedure? a. Clients level of anxiety b. Ability to turn self in bed c. Cardiac rhythm and heart rate d. Allergies to iodine-based agents

ANS: D Before the procedure, the nurse should ascertain whether the client has an allergy to iodine-containing preparations, such as seafood or local anesthetics. The contrast medium used during the procedure is iodine based. This allergy can cause a life-threatening reaction, so it is a high priority. Second, it is important for the nurse to assess anxiety, mobility, and baseline cardiac status

18. An emergency department nurse triages clients who present with chest discomfort. Which client should the nurse plan to assess first? a. A 42-year-old female who describes her pain as a dull ache with numbness in her fingers b. A 49-year-old male who reports moderate pain that is worse on inspiration c. A 53-year-old female who reports substernal pain that radiates to her abdomen d. A 58-year-old male who describes his pain as intense stabbing that spreads across his chest

ANS: D All clients who have chest pain should be assessed more thoroughly. To determine which client should be seen first, the nurse must understand common differences in pain descriptions. Intense stabbing, vise-like substernal pain that spreads through the clients chest, arms, jaw, back, or neck is indicative of a myocardial infarction. The nurse should plan to see this client first to prevent cardiac cell death. A dull ache with numbness in the fingers is consistent with anxiety. Pain that gets worse with inspiration is usually related to a pleuropulmonary problem. Pain that spreads to the abdomen is often associated with an esophageal-gastric problem, especially when this pain is experienced by a male client. Female clients may experience abdominal discomfort with a myocardial event. Although clients with anxiety, pleuropulmonary, and esophageal-gastric problems should be seen, they are not a higher priority than myocardial infarction.

11. A nursing student is struggling to understand the process of graft-versus-host disease. What explanation by the nurse instructor is best? a. Because of immunosuppression, the donor cells take over. b. Its like a transfusion reaction because no perfect matches exist. c. The clients cells are fighting donor cells for dominance. d. The donors cells are actually attacking the clients cells

ANS: D Graft versus host disease is an autoimmune-type process in which the donor cells recognize the clients cells as foreign and begin attacking them. The other answers are not accurate.

8. A nursing student is caring for a client with leukemia. The student asks why the client is still at risk for infection when the clients white blood cell count (WBC) is high. What response by the registered nurse is best? a. If the WBCs are high, there already is an infection present. b. The client is in a blast crisis and has too many WBCs. c. There must be a mistake; the WBCs should be very low. d. Those WBCs are abnormal and dont provide protection.

ANS: D In leukemia, the WBCs are abnormal and do not provide protection to the client against infection. The other statements are not accurate.

1. A nursing student wants to know why clients with chronic obstructive pulmonary disease tend to be polycythemic. What response by the nurse instructor is best? a. It is due to side effects of medications for bronchodilation. b. It is from overactive bone marrow in response to chronic disease. c. It combats the anemia caused by an increased metabolic rate. d. It compensates for tissue hypoxia caused by lung disease.

ANS: D In response to hypoxia, more red blood cells are made so more oxygen can be carried and delivered to tissues. This is a physiologic process in response to the disease; it is not a medication side effect, the result of overactive bone marrow, or a response to anemia.

30.The nurse is caring for a patient who suddenly experiences chest pain. What is the nurse's first priority? a. Call the rapid response team. b. Start an intravenous (IV) line. c. Administer pain-relief medications. d. Ask the patient to rate and describe the pain.

ANS: D The nurse's ability to establish a nursing diagnosis, plan and implement care, and evaluate the effectiveness of care depends on an accurate and timely assessment. The other responses are all interventions; the nurse cannot know which intervention is appropriate until the nurse completes the assessment.

25. A nurse is assessing a client who had a myocardial infarction. Upon auscultating heart sounds, the nurse hears the following sound. What action by the nurse is most appropriate? (Click the media button to hear the audio clip.) a. Assess for further chest pain. b. Call the Rapid Response Team. c. Have the client sit upright. d. Listen to the clients lung sounds.

ANS: D The sound the nurse hears is an S3 heart sound, an abnormal sound that may indicate heart failure. The nurse should next assess the clients lung sounds. Assessing for chest pain is not directly related. There is no indication that the Rapid Response Team is needed. Having the client sit up will not change the heart sound.

28. The nurse assesses a clients oral cavity and makes the discovery shown in the photo below: What action by the nurse is most appropriate? a. Encourage the client to have genetic testing. b. Instruct the client on high-fiber foods. c. Place the client in protective precautions. d. Teach the client about cobalamin therapy.

ANS: D This condition is known as glossitis, and is characteristic of B12 anemia. If the anemia is a pernicious anemia, it is treated with cobalamin. Genetic testing is not a priority for this condition. The client does not need highfiber foods or protective precautions.

9. A nurse prepares to defibrillate a client who is in ventricular fibrillation. Which priority intervention should the nurse perform prior to defibrillating this client? a. Make sure the defibrillator is set to the synchronous mode. b. Administer 1 mg of intravenous epinephrine. c. Test the equipment by delivering a smaller shock at 100 joules. d. Ensure that everyone is clear of contact with the client and the bed.

ANS: D To avoid injury, the rescuer commands that all personnel clear contact with the client or the bed and ensures their compliance before delivery of the shock. A precordial thump can be delivered when no defibrillator is available. Defibrillation is done in asynchronous mode. Equipment should not be tested before a client is defibrillated because this is an emergency procedure; equipment should be checked on a routine basis. Epinephrine should be administered after defibrillation.

12. The nurse is preparing to change a clients sternal dressing. What action by the nurse is most important? a. Assess vital signs. b. Don a mask and gown. c. Gather needed supplies. d. Perform hand hygiene.

ANS: D To prevent a sternal wound infection, the nurse washes hands or performs hand hygiene as a priority. Vital signs do not necessarily need to be assessed beforehand. A mask and gown are not needed. The nurse should gather needed supplies, but this is not the priority.

19. A nurse is preparing to hang a blood transfusion. Which action is most important? a. Documenting the transfusion b. Placing the client on NPO status c. Placing the client in isolation d. Putting on a pair of gloves

ANS: D To prevent bloodborne illness, the nurse should don a pair of gloves prior to hanging the blood. Documentation is important but not the priority at this point. NPO status and isolation are not needed.

15.A nurse is assessing a patient who started to have severe pain 3 days ago. When the nurse asks the patient to describe the pain, the patient states, "The pain feels like it is in my stomach. It is a burning pain, and it spreads out in a circle around the spot where it hurts the most." Which type of pain does the nurse document the patient is having at this time? a. Superficial pain b. Idiopathic pain c. Chronic pain d. Visceral pain

ANS: D Visceral pain arises from visceral organs, such as those from the gastrointestinal tract. Visceral pain is diffuse and radiates in several directions and can have a burning quality. Superficial pain has a short duration and is usually a sharp pain arising from the skin. Pain of an unknown cause is called idiopathic pain. Chronic pain lasts longer than 6 months.

16. A client with autoimmune idiopathic thrombocytopenic purpura (ITP) has had a splenectomy and returned to the surgical unit 2 hours ago. The nurse assesses the client and finds the abdominal dressing saturated with blood. What action is most important? a. Preparing to administer a blood transfusion b. Reinforcing the dressing and documenting findings c. Removing the dressing and assessing the surgical site d. Taking a set of vital signs and notifying the surgeon

ANS: D While some bloody drainage on a new surgical dressing is expected, a saturated dressing is not. This client is already at high risk of bleeding due to the ITP. The nurse should assess vital signs for shock and notify the surgeon immediately. The client may or may not need a transfusion. Reinforcing the dressing is an appropriate action, but the nurse needs to do more than document afterward. Removing the dressing increases the risk of infection; plus, it is not needed since the nurse knows where the bleeding is coming from.

2. A nurse assesses a client after administering a prescribed beta blocker. Which assessment should the nurse expect to find? a. Blood pressure increased from 98/42 mm Hg to 132/60 mm Hg b. Respiratory rate decreased from 25 breaths/min to 14 breaths/min c. Oxygen saturation increased from 88% to 96% d. Pulse decreased from 100 beats/min to 80 beats/min

ANS: D Beta blockers block the stimulation of beta1-adrenergic receptors. They block the sympathetic (fight-or-flight) response and decrease the heart rate (HR). The beta blocker will decrease HR and blood pressure, increasing ventricular filling time. It usually does not have effects on beta2-adrenergic receptor sites. Cardiac output will drop because of decreased HR.

1. The nurse is caring for a postoperative patient. The nurse will anticipate administering which medication to this patient to help prevent thrombus formation caused by slow venous blood flow? a. Alteplase (Activase) b. Aspirin c. Clopidogrel (Plavix) d. Lowmolecular-weight heparin

ANS: D Lowmolecular-weight heparin is an anticoagulant, which is used to inhibit clot formation and is used prophylactically to prevent postoperative deep vein thrombosis. Alteplase is a thrombolytic, which is used to break down clots after they form; alteplase is contraindicated in any patient with recent surgery. Aspirin and clopidogrel are antiplatelet drugs and are used to prevent arterial thrombosis.

1. A nurse assesses a clients electrocardiograph tracing and observes that not all QRS complexes are preceded by a P wave. How should the nurse interpret this observation? a. The client has hyperkalemia causing irregular QRS complexes. b. Ventricular tachycardia is overriding the normal atrial rhythm. c. The clients chest leads are not making sufficient contact with the skin. d. Ventricular and atrial depolarizations are initiated from different sites.

ANS: D Normal rhythm shows one P wave preceding each QRS complex, indicating that all depolarization is initiated at the sinoatrial node. QRS complexes without a P wave indicate a different source of initiation of depolarization. This finding on an electrocardiograph tracing is not an indication of hyperkalemia, ventricular tachycardia, or disconnection of leads.

7. The nurse is teaching a patient who will begin taking warfarin (Coumadin) for atrial fibrillation. Which statement by the patient indicates understanding of the teaching? a. I should eat plenty of green, leafy vegetables while taking this drug. b. I should take a nonsteroidal anti-inflammatory drug (NSAID) instead of acetaminophen for pain or fever. c. I will take cimetidine (Tagamet) to prevent gastric irritation and bleeding. d. I will tell my dentist that I am taking this medication.

ANS: D Patients taking warfarin should tell their dentists that they are taking the medication because of the increased risk for bleeding. Patients should avoid foods high in vitamin K, which can decrease the effects of warfarin. Patients should not take NSAIDs or cimetidine (Tagamet) because they can displace warfarin from proteinbinding sites.

8. A client is in the preoperative holding area prior to an emergency coronary artery bypass graft (CABG). The client is yelling at family members and tells the doctor to just get this over with when asked to sign the consent form. What action by the nurse is best? a. Ask the family members to wait in the waiting area. b. Inform the client that this behavior is unacceptable. c. Stay out of the room to decrease the clients stress levels. d. Tell the client that anxiety is common and that you can help.

ANS: D Preoperative fear and anxiety are common prior to cardiac surgery, especially in emergent situations. The client is exhibiting anxiety, and the nurse should reassure the client that fear is common and offer to help. The other actions will not reduce the clients anxiety.

11. One hour after receiving intravenous morphine sulfate, a patient reports generalized itching. The nurse assesses the patient and notes clear breath sounds, no rash, respirations of 14 breaths per minute, a heart rate of 68 beats per minute, and a blood pressure of 110/70 mm Hg. Which action will the nurse take? a. Administer naloxone to reverse opiate overdose. b. Have resuscitation equipment available at the bedside. c. Prepare an epinephrine injection in case of an anaphylactic reaction. d. Reassure the patient that this is a common side effect of this drug.

ANS: D Pruritis is a common opioid side effect and can be managed with diphenhydramine. Patients developing anaphylaxis will have urticaria and hypotension, and these patients will need epinephrine and resuscitation. Respiratory depression is a sign of morphine overdose, which will require naloxone

18. A nurse assesses a clients electrocardiogram (ECG) and observes the reading shown below: How should the nurse document this clients ECG strip? a. Ventricular tachycardia b. Ventricular fibrillation c. Sinus rhythm with premature atrial contractions (PACs) d. Sinus rhythm with premature ventricular contractions (PVCs)

ANS: D Sinus rhythm with PVCs has an underlying regular sinus rhythm with ventricular depolarization that sometimes precede atrial depolarization. Ventricular tachycardia and ventricular fibrillation rhythms would not have sinus beats present. Premature atrial contractions are atrial contractions initiated from another region of the atria before the sinus node initiates atrial depolarization.

5. A hospitalized client has a platelet count of 58,000/mm3. What action by the nurse is best? a. Encourage high-protein foods. b. Institute neutropenic precautions. c. Limit visitors to healthy adults. d. Place the client on safety precautions.

ANS: D With a platelet count between 40,000 and 80,000/mm3, clients are at risk of prolonged bleeding even after minor trauma. The nurse should place the client on safety precautions. High-protein foods, while healthy, are not the priority. Neutropenic precautions are not needed as the clients white blood cell count is not low. Limiting visitors would also be more likely related to a low white blood cell count.

2. A nurse teaches a client with a new permanent pacemaker. Which instructions should the nurse include in this clients teaching? (Select all that apply.) a. Until your incision is healed, do not submerge your pacemaker. Only take showers. b. Report any pulse rates lower than your pacemaker settings. c. If you feel weak, apply pressure over your generator. d. Have your pacemaker turned off before having magnetic resonance imaging (MRI). e. Do not lift your left arm above the level of your shoulder for 8 weeks.

ANS:A, B, E The client should not submerge in water until the site has healed; after the incision is healed, the client may take showers or baths without concern for the pacemaker. The client should be instructed to report changes in heart rate or rhythm, such as rates lower than the pacemaker setting or greater than 100 beats/min. The client should be advised of restrictions on physical activity for 8 weeks to allow the pacemaker to settle in place. The client should never apply pressure over the generator and should avoid tight clothing. The client should never have MRI because, whether turned on or off, the pacemaker contains metal. The client should be advised to inform all health care providers that he or she has a pacemaker.

3. A nurse assesses clients on a cardiac unit. Which clients should the nurse identify as at greatest risk for the development of acute pericarditis? (Select all that apply.) a. A 36-year-old woman with systemic lupus erythematosus (SLE) b. A 42-year-old man recovering from coronary artery bypass graft surgery c. A 59-year-old woman recovering from a hysterectomy d. An 80-year-old man with a bacterial infection of the respiratory tract e. An 88-year-old woman with a stage III sacral ulcer

ANS:A,B,D Acute pericarditis is most commonly associated acute exacerbations of systemic connective tissue disease, including SLE; with Dresslers syndrome, or inflammation of the cardiac sac after cardiac surgery or a myocardial infarction; and with infective organisms, including bacterial, viral, and fungal infections. Abdominal and reproductive surgeries and pressure ulcers do not increase clients risk for acute pericarditis

1. A nursing student learns about modifiable risk factors for coronary artery disease. Which factors does this include? (Select all that apply.) a. Age b. Hypertension c. Obesity d. Smoking e. Stress

ANS:B, C, D, E Hypertension, obesity, smoking, and excessive stress are all modifiable risk factors for coronary artery disease. Age is a nonmodifiable risk factor.

20. A nurse assesses a client who has aortic regurgitation. In which location in the illustration shown below should the nurse auscultate to best hear a cardiac murmur related to aortic regurgitation?

The aortic valve is auscultated in the second intercostal space just to the right of the sternum.

Which client(s) are appropriate to assign to the LPN/LVN, who will function under the supervision of the RN or team leader? (Choose all that apply.) a.A client who needs pre-op teaching for use of a PCA pump b.A client with a leg cast who needs neurologic checks and PRN hydrocodone c.A client post-op toe amputation with diabetic neuropathic pain d.A client with terminal cancer and severe pain who is refusing medication

b.A client with a leg cast who needs neurologic checks and PRN hydrocodone c.A client post-op toe amputation with diabetic neuropathic pain The clients with the cast and the toe amputation are stable clients and need ongoing assessment and pain management that are within the scope of practice for an LPN/LVN under the supervision of an RN. The RN should take responsibility for pre-operative teaching, and the terminal cancer needs a comprehensive assessment to determine the reason for refusal of medication.

A family member asks you, "Why can't you give more medicine? He is still having a lot of pain." What is your best response? a."The doctor ordered the medicine to be given every 4 hours." b."If the medication is given too frequently he could suffer ill effects." c."Please tell him that I will be right there to check of him." d."Let's wait about 30-40 minutes. If there is no relief I'll call the doctor."

c. Please tell him that I will be right there to check of him directly ask the client about the pain and do a complete pain assessment. This information will determine which action to take next.

9. When caring for a patient who is receiving epidural morphine, which information obtained by the nurse indicates that the patient may be experiencing a side effect of the medication? a. The patient has cramping abdominal pain. b. The patient becomes restless and agitated. c. The patient has not voided for over 10 hours. d. The patient complains of a "pounding" headache.

c. The patient has not voided for over 10 hours. Urinary retention is a common side effect of epidural opioids. Headache is not an anticipated side effect of morphine, although if there is a cerebrospinal fluid leak, the patient may develop a "spinal" headache. Sedation (rather than restlessness or agitation) would be a possible side effect. Hypotonic bowel sounds and constipation (rather than abdominal cramping) are concerns.

13. The nurse is caring for a diabetic patient who has chronic burning leg pain even when taking oxycodone (OxyContin) twice daily. Which of these prescribed medications is the best choice for the nurse to administer as an adjuvant to decrease the patient's pain? a. aspirin (Ecotrin) b. celecoxib (Celebrex) c. amitriptyline (Elavil) d. acetaminophen (Tylenol)

c. amitriptyline (Elavil) The patient's pain symptoms are consistent with neuropathic pain and the tricyclic antidepressants are effective for treating this type of pain. The other medications are more effective for nociceptive pain.

12. Which of these prescribed therapies should the nurse use first when caring for a patient with cancer pain that the patient describes as at "level 8 (0 to 10 scale), deep, and aching." a. fentanyl (Duragesic) patch b. ketorolac (Toradol) tablets c. hydromorphone (Dilaudid) IV d. acetaminophen (Tylenol) suppository

c. hydromorphone (Dilaudid) IVThe patient's pain level indicates that a rapidly-acting medication such as an IV opioid is needed. The other medications also may be appropriate to use, but will not work as rapidly or as effectively as the IV hydromorphone.

7. A patient with chronic back pain is seen in the pain clinic for follow-up. In order to evaluate whether the pain management is effective, which question is best for the nurse to ask? a. "Can you describe the quality of your pain?" b. "Has there been a change in the pain location?" c. "How would you rate your pain on a 0 to 10 scale?" d. "Does the pain keep you from doing things you enjoy?"

d. "Does the pain keep you from doing things you enjoy?" The goal for the treatment of chronic pain usually is to enhance function and quality of life. The other questions also are appropriate to ask, but information about patient function is more useful in evaluating effectiveness.

4. A patient with chronic cancer pain is receiving imipramine (Tofranil) in addition to long-acting morphine for pain control. Which information is the best indicator that the imipramine is effective? a. The patient sleeps 8 hours every night. b. The patient has no symptoms of anxiety. c. The patient states, "I feel much less depressed since I've been taking the imipramine." d. The patient states, "The pain is manageable, and I can accomplish my desired activities.

d. The patient states, "The pain is manageable, and I can accomplish my desired activities. Imipramine is being used in this patient to manage chronic pain and improve functional ability. Although the medication also is prescribed for patients with depression, insomnia, and anxiety, the evaluation for this patient is based on improved pain control and activity level.

Which client is at greater risk for respiratory depression while receiving opioids for analgesia? a.An elderly chronic pain client with a hip fracture b.A client with a heroin addiction and back pain c.A young female client with advanced multiple myeloma d.A child with an arm fracture and cystic fibrosis

d.A child with an arm fracture and cystic fibrosis at greatest risk are elderly clients, opiate naïve clients, and those with underlying pulmonary disease. The child has two of the three risk factors

A client with chronic pain reports to you, the charge nurse, that the nurse have not been responding to requests for pain medication. What is your initial action? a.Check the MARs and nurses' notes for the past several days. b.Ask the nurse educator to give an in-service about pain management. c.Perform a complete pain assessment and history on the client. d.Have a conference with the nurses responsible for the care of this client

d.Have a conference with the nurses responsible for the care of this client As charge nurse, you must assess for the performance and attitude of the staff in relation to this client. After gathering data from the nurses, additional information from the records and the client can be obtained as necessary. The educator may be of assistance if knowledge deficit or need for performance improvement is the problem.


Ensembles d'études connexes

human growth and development final review

View Set

SOCI-1020 Chapter 3 & 4 Test (Kamolnick)

View Set

Religion Ch. 5 - Conquest and Exile: A Remnant Returns

View Set

Life Insurance Policy Provisions, Options and Riders

View Set

Prep U Chapter 13: Fluid and Electrolytes: Balance and Disturbance

View Set

Chapter 1: Discovering the Night Sky (Textbook Notes)

View Set